1 2 и 3 закон ньютона: первый, второй, третий закон кратко с объяснением, формулами

Содержание

первый, второй, третий закон кратко с объяснением, формулами

Мы уже говорили об основах классической механики. Настала пора поговорить о них подробнее и затронуть в обсуждении чуть больше, чем просто основу. В этой статье мы подробно разберем основные законы классической механики. Как вы уже догадались, речь пойдет о законах Ньютона.

Ежедневная рассылка с полезной информацией для студентов всех направлений – на нашем телеграм-канале.

Основные законы классической механики Исаак Ньютон (1642-1727) собрал и опубликовал в 1687 году. Три знаменитых закона были включены в труд, который назывался «Математические начала натуральной философии».

Был долго этот мир глубокой тьмой окутан
Да будет свет, и тут явился Ньютон.

(Эпиграмма 18-го века)

Но сатана недолго ждал реванша –
Пришел Эйнштейн, и стало все как раньше.

(Эпиграмма 20-го века)

Что стало, когда пришел Эйнштейн, читайте в отдельном материале про релятивистскую динамику.

А мы пока приведем формулировки и примеры решения задач на каждый закон Ньютона.

Первый закон Ньютона

Первый закон Ньютона гласит:

Существуют такие системы отсчета, называемые инерциальными, в которых тела движутся равномерно и прямолинейно, если на них не действуют никакие силы или действие других сил скомпенсировано.

Проще говоря, суть первого закона Ньютона можно сформулировать так: если мы на абсолютно ровной дороге толкнем тележку и представим, что можно пренебречь силами трения колес и сопротивления воздуха, то она будет катиться с одинаковой скоростью бесконечно долго.

Инерция – это способность тела сохранять скорость как по направлению, так и по величине, при отсутствии воздействий на тело. Первый закон Ньютона еще называют законом инерции.

До Ньютона закон инерции был сформулирован в менее четкой форме Галилео Галилеем. Инерцию ученый называл «неистребимо запечатленным движением». Закон инерции Галилея гласит: при отсутствии внешних сил тело либо покоится, либо движется равномерно. Огромная заслуга Ньютона в том, что он сумел объединить принцип относительности Галилея, собственные труды и работы других ученых в своих “Математических началах натуральной философии”.

Понятно, что таких систем, где тележку толкнули, а она покатилась без действия внешних сил, на самом деле не бывает. На тела всегда действуют силы, причем скомпенсировать действие этих сил полностью практически невозможно.

Например, все на Земле находится в постоянном поле силы тяжести. Когда мы передвигаемся (не важно, ходим пешком, ездим на машине или велосипеде), нам нужно преодолевать множество сил: силу трения качения и силу трения скольжения, силу тяжести, силу Кориолиса.

 

Второй закон Ньютона

Помните пример про тележку? В этот момент мы приложили к ней силу! Интуитивно понятно, что тележка покатится и вскоре остановится. Это значит, ее скорость изменится.

В реальном мире скорость тела чаще всего изменяется, а не остается постоянной. Другими словами, тело движется с ускорением. Если скорость нарастает или убывает равномерно, то говорят, что движение равноускоренное.

Если рояль падает с крыши дома вниз, то он движется равноускоренно под действием постоянного ускорения свободного падения g. Причем любой дугой предмет, выброшенный из окна на нашей планете, будет двигаться с тем же ускорением свободного падения.

Второй закон Ньютона устанавливает связь между массой, ускорением и силой, действующей на тело. Приведем формулировку второго закона Ньютона:

Ускорение тела (материальной точки) в инерциальной системе отсчета прямо пропорционально приложенной к нему силе и обратно пропорционально массе.

 

Если на тело действует сразу несколько сил, то в данную формулу подставляется равнодействующая всех сил, то есть их векторная сумма.

В такой формулировке второй закон Ньютона применим только для движения со скоростью, много меньшей, чем скорость света.

Существует более универсальная формулировка данного закона,  так называемый дифференциальный вид.

В любой бесконечно малый промежуток времени dt сила, действующая на тело, равна производной импульса тела по времени.

Третий закон Ньютона

В чем состоит третий закон Ньютона? Этот закон описывает взаимодействие тел.

3 закон Ньютона говорит нам о том, что на любое действие найдется противодействие. Причем, в прямом смысле:

Два тела воздействуют друг на друга с силами, противоположными по направлению, но равными по модулю.

Формула, выражающая третий закон Ньютона:

Другими словами, третий закон Ньютона – это закон действия и противодействия.

 

Пример задачи на законы Ньютона

Вот типичная задачка на применение законов Ньютона. В ее решении используются первый и второй законы Ньютона.

Десантник раскрыл парашют и опускается вниз с постоянной скоростью. Какова сила сопротивления воздуха? Масса десантника – 100 килограмм.

Решение:  

Движение парашютиста – равномерное и прямолинейное, поэтому, по первому закону Ньютона, действие сил на него скомпенсировано.

На десантника действуют сила тяжести и сила сопротивления воздуха. Силы направлены в противоположные стороны.

По второму закону Ньютона, сила тяжести равна ускорению свободного падения, умноженному на массу десантника.

Ответ: Сила сопротивления воздуха равна силе тяжести по модулю и противоположна направлена.

Кстати! Для наших читателей сейчас действует скидка 10% на любой вид работы

А вот еще одна физическая задачка на понимание действия третьего закона Ньютона.

Комар ударяется о лобовое стекло автомобиля. Сравните силы, действующие на автомобиль и комара.

Решение:

По третьему закону Ньютона, силы, с которыми тела действуют друг на друга, равны по модулю и противоположны по направлению. Сила, с которой комар действует на автомобиль, равна силе, с которой автомобиль действует на комара.

Другое дело, что действие этих сил на тела сильно отличаются вследствие различия масс и ускорений.

Исаак Ньютон: мифы и факты из жизни

На момент публикации своего основного труда Ньютону было 45 лет. За свою долгую жизнь ученый внес огромный вклад в науку, заложив фундамент современной физики и определив ее развитие на годы вперед.

Он занимался не только механикой, но и оптикой, химией и другими науками, неплохо рисовал и писал стихи. Неудивительно, что личность Ньютона окружена множеством легенд.

Ниже приведены некоторые факты и мифы из жизни И. Ньютона. Сразу уточним, что миф – это не достоверная информация. Однако мы допускаем, что мифы и легенды не появляются сами по себе и что-то из перечисленного вполне может оказаться правдой.

  • Факт. Исаак Ньютон был очень скромным и застенчивым человеком. Он увековечил себя благодаря своим открытиям, однако сам никогда не стремился к славе и даже пытался ее избежать.
  • Миф. Существует легенда, согласно которой Ньютона осенило, когда на наго в саду упало яблоко. Это было время чумной эпидемии (1665-1667), и ученый был вынужден покинуть Кембридж, где постоянно трудился. Точно неизвестно, действительно ли падение яблока было таким роковым для науки событием, так как первые упоминания об этом появляются только в биографиях ученого уже после его смерти, а данные разных биографов расходятся.
  • Факт. Ньютон учился, а потом много работал в Кембридже. По долгу службы ему нужно было несколько часов в неделю вести занятия у студентов. Несмотря на признанные заслуги ученого, занятия Ньютона посещались плохо. Бывало, что на его лекции вообще никто не приходил. Скорее всего, это связано с тем, что ученый был полностью поглощен своими собственными исследованиями.
  • Миф. В 1689 году Ньютон был избран членом Кембриджского парламента. Согласно легенде, более чем за год заседания в парламенте вечно поглощенный своими мыслями ученый взял слово для выступления всего один раз.
    Он попросил закрыть окно, так как был сквозняк.
  • Факт. Неизвестно, как бы сложилась судьба ученого и всей современной науки, если бы он послушался матери и начал заниматься хозяйством на семейной ферме. Только благодаря уговорам учителей и своего дяди юный Исаак отправился учиться дальше вместо того, чтобы сажать свеклу, разбрасывать по полям навоз и по вечерам выпивать в местных пабах.

Дорогие друзья, помните – любую задачу можно решить! Если у вас возникли проблемы с решением задачи по физике, посмотрите на основные физические формулы. Возможно, ответ перед глазами, и его нужно просто рассмотреть. Ну а если времени на самостоятельные занятия совершенно нет, специализированный студенческий сервис всегда к вашим услугам!

В самом конце предлагаем посмотреть видеоурок на тему “Законы Ньютона”.

Знаменитые законы Ньютона

4 января – это особая дата в научном обществе, ведь именно в этот день появился на свет Исаак Ньютон. О том, как его законы действуют в “Артеке”, читайте в статье медиаотряда д/л “Янтарный”

4 января весь мир отмечает День Ньютона. 
Не слышали о таком празднике? Сейчас мы вам расскажем!

Кто такой Исаак Ньютон? Это выдающийся английский ученый, разработавший и открывший ряд важнейших для мировой науки законов и теорий в физике, математике и астрономии. Многие из его открытий входят в школьную программу для старших классов.

Самый знаменитый закон Ньютона — это закон всемирного тяготения: Два любых тела притягиваются друг к другу с силой, прямо пропорциональной массе каждого из них и обратно пропорциональной квадрату расстояния между ними. 
 


                 Каждую смену артековцы обязательно притягиваются к вкусной анимации.

Второй закон Ньютона: Ускорение тела прямо пропорционально равнодействующей всех сил, приложенных к телу, и обратно пропорционально его массе.

Приходя к Дереву Желаний, каждый артековец хочет, чтобы его мечта сбылась намного скорее.

Третий закон Ньютона гласит: Взаимодействия двух тел друг на друга равны между собой и направлены в противоположные стороны.

Интересный факты:

– Хотя многоцветный спектр радуги непрерывен, по традиции в нём выделяют семь цветов. Считается, что первым выбрал число семь Исаак Ньютон. Причём первоначально, он различал только пять цветов – красный, жёлтый, зелёный, голубой и фиолетовый, о чём и написал в своей книге ”Оптика”.

– Благодаря Ньютону сократилось создание фальшивых монет, так как он сообразил по бокам делать линии, которые предотвращали срезание металла.

– На первом логотипе Apple был изображён Исаак, который сидел под яблоней.

Авторы: медиаотряд д/л «Янтарный», 15 смена 2019 года

Три закона Ньютона | Физика

Раздел механики, в котором изучают, как взаимодействие тел влияет на их движение, называют динамикой.

Основные законы динамики открыли итальянский ученый Галилео Галилей и английский ученый Исаак Ньютон. Вы изучали эти законы в курсе физики основной школы. Напомним их.

1. Первый закон ньютона (закон инерции)

Повторим один из опытов, которые поставил итальянский ученый Галилео Галилей.

Поставим опыт
Будем скатывать шар по наклонной плоскости и наблюдать за его дальнейшим движением по горизонтальной поверхности.
Если она посыпана песком, шар остановится очень скоро (рис. 13.1, а).
Если она покрыта тканью, шар катится значительно дольше (рис. 13.1, б).
А вот по стеклу шар катится очень долго (рис. 13.1, в).

На основании этого и подобных опытов Галилей открыл закон инерции: если на тело не действуют другие тела или действия других тел скомпенсированы, то тлело движется равномерно и прямолинейно или покоится.

Сохранение скорости тела, когда на него не действуют другие тела или действия других тел скомпенсированы, называют явлением инерции.

? 1. Почему при встряхивании мокрого зонта с него слетают капли воды?

Особенно красиво смотрится явление инерции в фигурном катании (рис. 13.2).

Закон инерции называют также первым законом Ньютона, потому что Ньютон включил его в качестве первого закона в систему трех законов динамики, которые называют «тремя законами Ньютона».

Инерциальные системы отсчета

Закон инерции выполняется с хорошей точностью в системе отсчета, связанной с Землей. Но он не выполняется, например, в системе отсчета, связанной с тормозящим автобусом: при резком торможении пассажиры отклоняются вперед, хотя на них не действуют направленные вперед силы.
Системы отсчета, в которых выполняется закон инерции, называют инерциальными.

Инерциальных систем отсчета бесконечно много. Ведь если некоторая система отсчета является инерциальной, то инерциальной будет любая другая система отсчета, движущаяся относительно нее прямолинейно и равномерно.

Сформулируем теперь первый закон Ньютона с указанием систем отсчета, в которых он выполняется.

Существуют системы отсчета (называемые инерциальными), относительно которых тела сохраняют свою скорость неизменной, если на них не действуют другие тела или действия других тел скомпенсированы.

Изучать влияние взаимодействия тел на их движение удобнее всего именно в инерциальных системах отсчета, потому что в этих системах отсчета изменение скорости тела обусловлено только действием других тел на это тело.

Принцип относительности Галилея

Как показывает опыт, во всех инерциальных системах отсчета все механические явления протекают одинаково при одинаковых начальных условиях.

Это утверждение называют принципом относительности Галилея.

В справедливости принципа относительности Галилея легко убедиться, сидя в поезде, который плавно движется с постоянной скоростью. В таком случае все опыты с механическими явлениями, поставленные в вагоне, дадут одинаковые результаты независимо от того, едет поезд или стоит: например, лежащее на столе яблоко будет покоиться, а свободно падающие предметы будут падать вертикально вниз (относительно вагона!).

Поэтому пассажир может определить, едет поезд или стоит на станции, только посмотрев в окно (рис. 13.3).

2. Второй закон ньютона

Равнодействующая

Как вы уже знаете из курса физики основной школы, силы – векторные величины: каждая сила характеризуется числовым значением (модулем) и направлением. Силы измеряют с помощью динамометров. Единицей силы в СИ является 1 ньютон (Н). Определение ньютона мы дадим позже.

Если на тело, которое можно считать материальной точкой, действуют несколько сил, то их можно заменить одной силой, которая является векторной суммой этих сил. Ее называют равнодействующей.

На рисунке 13.4 показано, как найти равнодействующую двух сил: а

? 2. К телу приложены две силы, равные по модулю 1 Н и 2 Н. Отвечая на следующие вопросы, сделайте пояснительные чертежи.
а) Какое наименьшее значение может принимать равнодействующая этих сил? Как направлены силы в этом случае?
б) Какое наибольшее значение может быть у равнодействующей этих сил? Как направлены силы в атом случае?
в) Может ли равнодействующая этих сил быть равной 2 Н?

? 3. К телу приложены две силы, равные по модулю 3 Н и 4 Н. Может ли их равнодействующая быть равной 5 Н? Если да, то чему в этом случае равен угол между приложенными силами?

? 4. К телу приложены три равные по модулю силы по 1 Н каждая. Как они должны быть направлены, чтобы:
а) равнодействующая была равна 1 Н?
б) равнодействующая была равна нулю?
в) равнодействующая была равна 2 Н?

Масса тела

В курсе физики основной школы рассказывалось также об опытах, которые доказывают, что под действием постоянной силы тело движется с постоянным ускорением.

Коэффициент пропорциональности между силой и ускорением характеризует инертные свойства тела и называется массой тела. Чем больше масса тела, тем большую силу надо приложить к телу, чтобы сообщить ему то же ускорение.

Единицей массы в СИ является 1 килограмм (кг). Это масса эталона, хранящегося в Международном бюро мер и весов (Франция). Приближенно можно считать, что одному килограмму равна масса 1 л воды.

Обозначают массу буквой m.

Второй закон Ньютона

Соотношение между равнодействующей всех сил, приложенных к телу, массой тела и его ускорением Ньютон сформулировал как второй из трех основных законов механики.

Равнодействующая всех сил, приложенных к телу, равна произведению массы тела на его ускорение:

В инерциальной системе отсчета сила является причиной ускорения, поэтому второй закон Ньютона часто записывают так:

Итак, приобретаемое телом ускорение прямо пропорционально равнодействующей приложенных к телу сил, одинаково с ней направлено и обратно пропорционально массе тела.

Заметим, что второй закон Ньютона справедлив только в инерциальных системах отсчета. Напомним: в этих системах отсчета ускорение тела обусловлено только действием на него других тел.

Единицу силы в СИ определяют на основе второго закона Ньютона: сила в 1 ньютон сообщает телу массой 1 кг ускорение 1 м/с2. Поэтому 1 Н = 1 кг * м/с2.

Сила тяжести

Как вы уже знаете, под действием притяжения Земли все тела падают с одинаковым ускорением – ускорением свободного падения . Силу притяжения, действующую на тело со стороны Земли, называют силой тяжести и обозначают т.

Когда тело свободно падает, на него действует только сила тяжести, поэтому она и является равнодействующей всех приложенных к телу сил. При атом тело движется с ускорением , поэтому из второго закона Ньютона получаем:

? 5. С какой силой Земля притягивает:
а) килограммовую гирю?
б) человека массой 60 кг?

Сила, скорость и ускорение – кто «третий лишний»?

Неочевидное следствие второго закона Ньютона состоит в том, что он утверждает: направление ускорения тела совпадает с направлением равнодействующей приложенных телу сил. Скорость же вела может быть при этом направлена как угодно!

Поставим опыт

Бросим шарик вниз, затем – вверх, а потом – под углом к горизонту (рис. 13.5)

На шарик во время всего движения действует только направленная вниз сила тяжести. Однако в первом случае (а) скорость шарика совпадает по направлению с этой силой, во втором случае (б) – скорость вначале противоположна силе тяжести, а в третьем (в) – скорость направлена под углом к силе тяжести (например, в верхней точке траектории скорость перпендикулярна силе тяжести).

? 6. Тело равномерно движется по окружности. Чему равен угол между скоростью тела и равнодействующей?

? 7. Чему равен угол между скоростью автомобиля и равнодействующей приложенных к нему сил, когда автомобиль:
а) разгоняется на прямой дороге?
б) тормозит на прямой дороге?
в) движется равномерно по дуге окружности?

3. Третий закон ньютона

Поставим опыт

Предложим первокласснику и десятикласснику посоревноваться в перетягивании каната, стоя на скейтбордах: тогда трением между колесами и полом можно пренебречь (схема опыта показана на рисунке 13.6).

Мы увидим, что оба соперника движутся с ускорением. Значит, на каждого из них действу другого. Ускорения соперников направлено противоположно, причем ускорение первоклассника намного больше ускорения десятиклассника.

Точные опыты, подобные описанном выше, показывают, что модули ускорений обратно пропорциональны массам тел:

a1/a2 = m2/m1.

Поскольку ускорения направлены противоположно,

Согласно второму закону Ньютона m11 = 1 и m22 = 2, где 1 – сила, действующая на первое тело со стороны второго, а 2 – сила, действующая на второе тело со стороны первого.

Из соотношения (5) следует, что 1 = –2. Это и есть третий закон Ньютона.

Тела взаимодействуют друг с другом с силами, равными по модулю и противоположными по направлению.

Свойстве сил, с которыми тела взаимодействуют друг с другом:
– эти силы обусловлены одним и тем же взаимодействием и поэтому имеют одну и ту же физическую природу;
– эти силы направлены вдоль одной прямой;
– эти силы приложены к разным телам и поэтому не могут уравновешивать друг друга.

Примеры проявления третьего закона Ньютона

Когда камень падает на Землю, на него действует сила тяжести 1 со стороны Земли, а на Землю – сила 2 притяжения со стороны камня (рис. 13.7, для наглядности масштаб не соблюден). Обе эти силы относятся к силам всемирного тяготения.

? 8. Согласно третьему закону Ньютона F1 = F2. Почему же ускорение камня заметно, а ускорение Земли – нет?

Когда камень лежит на Земле, на него кроме силы тяжести, которую будем обозначать теперь т, действует еще направленная вверх сила давления со стороны опоры (рис. 13.8, а). Она направлена перпендикулярно поверхности опоры, поэтому ее называют силой нормальной реакции (перпендикуляр называют часто нормалью). (Когда тело можно считать материальной точкой, все действующие на него силы желательно изображать на чертежах приложенными в одной точке.)

Когда камень покоится, его ускорение равно нулю. Значит, согласно второму закону Ньютона равнодействующая приложенных к камню сил и т, равна нулю (будем говорить, что в таком случае силы уравновешивают друг друга):

Отсюда следует:

Опора давит на камень силой , направленной вверх, а камень, по третьему закону Ньютона, давит на опору силан , направленной вниз (рис. 13.8, 6). Обе эти силы – силы упругости.

Силу, с которой тело вследствие действия на него силы тяжести давит на горизонтальную опору или растягивает вертикальный поднес, называют весом тела.

Итак, – это вес камня. По третьему закону Ньютона

Из формул (8) и (9) следует:

Итак, вес покоящегося тела равен действующей на это тело силе тяжести. Однако несмотря на это вес и сила тяжести существенно отличаются друг от друга:
– эти силы приложены к разным телам: вес действует на опору или поднес, а сила тяжести – на само тело;
– эти силы имеют разную физическую природу: вес – это сила упругости, а сила тяжести – проявление сил всемирного тяготения.

Кроме того, как мы увидим несколько позже (§ 16), вес может быть не равен силе тяжести и даже быть равным нулю.

Дополнительные вопросы и задания

9. Ускорение тела в некоторой инерциальной системе отсчета равно 3 м/с2 и направлено вдоль оси x. Чему равно ускорение этого тела в инерциальной системе отсчета, движущейся относительно заданной со скоростью 4 м/с, направленной вдоль оси y? Есть ли здесь лишние данные?

10. Брусок массой 0,5 кг соскальзывает с наклонной плоскости с углом наклона 30º. Скорость бруска увеличивается. Ускорение бруска равно 2 м/с2. Изобразите на чертеже равнодействующую приложенных к бруску сил. Чему она равна? Есть ли в задаче лишние данные?

11. Зависимость координаты x автомобиля от времени выражается в единицах СИ формулой x = 20 – 10t + t2. Ось x направлена вдоль дороги, масса автомобиля 1 т.
а) Чему равна равнодействующая приложенных к автомобилю сил?
б) Как она направлена в начальный момент – в направлении скорости автомобиля или противоположно ей?

12. Автомобиль массой 1 т едет со скоростью 72 км/ч по выпуклому мосту, имеющему форму дуги окружности радиусом 50 м. Сделайте чертеж и ответьте на вопросы.
а) Чему равна и как направлена равнодействующая сил, приложенных к автомобилю в верхней точке моста?
б) Какие силы действуют на автомобиль в этой точке? Как они направлены и чему они равны?
в) Во сколько раз вес автомобиля в верхней точке моста меньше действующей на автомобиль силы тяжести?

Физики изучили условия, при которых не работает третий закон Ньютона

https://ria. ru/20200818/1575917036.html

Физики изучили условия, при которых не работает третий закон Ньютона

Физики изучили условия, при которых не работает третий закон Ньютона – РИА Новости, 18.08.2020

Физики изучили условия, при которых не работает третий закон Ньютона

Российские ученые из Объединенного института высоких температур (ОИВТ) РАН и лаборатории активных сред и систем МФТИ совместно с американскими коллегами… РИА Новости, 18.08.2020

2020-08-18T03:27

2020-08-18T03:27

2020-08-18T07:41

наука

россия

физика

российский научный фонд

московский физико-технический институт

российская академия наук

сша

/html/head/meta[@name=’og:title’]/@content

/html/head/meta[@name=’og:description’]/@content

https://cdn23.img.ria.ru/images/155936/84/1559368462_0:236:2801:1812_1920x0_80_0_0_afbaa0c10ee8fdb9376f31d85caa92d8.jpg

МОСКВА, 18 авг — РИА Новости. Российские ученые из Объединенного института высоких температур (ОИВТ) РАН и лаборатории активных сред и систем МФТИ совместно с американскими коллегами экспериментально получили и детально исследовали систему взаимодействующих частиц, для которых формально не выполняется третий закон Ньютона, рассказали РИА Новости в пресс-службе ОИВТ. “Третий закон Ньютона, который все помнят из школьной программы, утверждает, что сила действия равняется силе противодействия. Однако для некоторых открытых и неравновесных дисперсных систем — частиц в среде — симметрия эффективной силы межчастичного взаимодействия может нарушаться, и возникает очень интересная физика: например, частицы самоорганизуются в сложные структуры, система аномально разогревается, появляются необычные неравновесные фазовые переходы”, — отметил заведующий лабораторией диагностики пылевой плазмы ОИВТ Евгений Лисин.Впервые систему с несимметричным взаимодействием частиц удалось получить в конце 1990-х годов в Германии. Но с тех пор, несмотря на подготовленную теоретическую базу, одной из важных нерешенных проблем оставалось прямое экспериментальное исследование особенностей несимметричного взаимодействия между частицами. “Точно измерить силу межчастичного взаимодействия и определить степень нарушения симметрии в зависимости от условий среды ранее не удавалось”, — рассказали в пресс-службе ОИВТ. Решение этой проблемы стало возможным благодаря оригинальному спектральному методу измерения, который был разработан отечественными специалистами при поддержке Российского научного фонда.Как отметили в ОИВТ, нарушения симметрии межчастичного взаимодействия имеют общие закономерности с поведением колоний бактерий, косяков рыб, стай птиц и т.д. Данное направление исследований также может быть интересно в контексте разработки новых материалов с “программируемым” откликом на механические напряжения, магнитные и тепловые поля. “Появляются также перспективные приложения, связанные с сепарацией вещества, коллективной адресной доставкой микрогруза (например, лекарств) и преобразованием механической энергии хаотического движения”, — рассказали в пресс-службе.Научная статья “Экспериментальное исследование несимметричных взаимодействий между микрочастицами в анизотропной плазме” опубликована российскими учеными в журнале Scientific Reports в соавторстве с американскими физиками из астрофизического центра CASPER Бейлорского университета.

https://ria.ru/20200817/1575904769.html

https://ria.ru/20200731/1575205075.html

россия

сша

РИА Новости

[email protected]

7 495 645-6601

ФГУП МИА «Россия сегодня»

https://xn--c1acbl2abdlkab1og.xn--p1ai/awards/

2020

РИА Новости

[email protected]

7 495 645-6601

ФГУП МИА «Россия сегодня»

https://xn--c1acbl2abdlkab1og.xn--p1ai/awards/

Новости

ru-RU

https://ria.ru/docs/about/copyright.html

https://xn--c1acbl2abdlkab1og.xn--p1ai/

РИА Новости

[email protected]

7 495 645-6601

ФГУП МИА «Россия сегодня»

https://xn--c1acbl2abdlkab1og.xn--p1ai/awards/

https://cdn22.img.ria.ru/images/155936/84/1559368462_36:0:2767:2048_1920x0_80_0_0_d19f807bc728e2d75a215f8302a54681.jpg

РИА Новости

[email protected]

7 495 645-6601

ФГУП МИА «Россия сегодня»

https://xn--c1acbl2abdlkab1og. xn--p1ai/awards/

РИА Новости

[email protected]

7 495 645-6601

ФГУП МИА «Россия сегодня»

https://xn--c1acbl2abdlkab1og.xn--p1ai/awards/

россия, физика, российский научный фонд, московский физико-технический институт, российская академия наук, сша

МОСКВА, 18 авг — РИА Новости. Российские ученые из Объединенного института высоких температур (ОИВТ) РАН и лаборатории активных сред и систем МФТИ совместно с американскими коллегами экспериментально получили и детально исследовали систему взаимодействующих частиц, для которых формально не выполняется третий закон Ньютона, рассказали РИА Новости в пресс-службе ОИВТ.

“Третий закон Ньютона, который все помнят из школьной программы, утверждает, что сила действия равняется силе противодействия. Однако для некоторых открытых и неравновесных дисперсных систем — частиц в среде — симметрия эффективной силы межчастичного взаимодействия может нарушаться, и возникает очень интересная физика: например, частицы самоорганизуются в сложные структуры, система аномально разогревается, появляются необычные неравновесные фазовые переходы”, — отметил заведующий лабораторией диагностики пылевой плазмы ОИВТ Евгений Лисин.

17 августа 2020, 18:00НаукаАстрономы зафиксировали необычное “сердцебиение” черной дыры

Впервые систему с несимметричным взаимодействием частиц удалось получить в конце 1990-х годов в Германии. Но с тех пор, несмотря на подготовленную теоретическую базу, одной из важных нерешенных проблем оставалось прямое экспериментальное исследование особенностей несимметричного взаимодействия между частицами. “Точно измерить силу межчастичного взаимодействия и определить степень нарушения симметрии в зависимости от условий среды ранее не удавалось”, — рассказали в пресс-службе ОИВТ.

Решение этой проблемы стало возможным благодаря оригинальному спектральному методу измерения, который был разработан отечественными специалистами при поддержке Российского научного фонда.

Как отметили в ОИВТ, нарушения симметрии межчастичного взаимодействия имеют общие закономерности с поведением колоний бактерий, косяков рыб, стай птиц и т.д. Данное направление исследований также может быть интересно в контексте разработки новых материалов с “программируемым” откликом на механические напряжения, магнитные и тепловые поля. “Появляются также перспективные приложения, связанные с сепарацией вещества, коллективной адресной доставкой микрогруза (например, лекарств) и преобразованием механической энергии хаотического движения”, — рассказали в пресс-службе.

31 июля 2020, 11:05НаукаУченые выяснили, что Вселенная более однородная, чем ожидалось

Научная статья “Экспериментальное исследование несимметричных взаимодействий между микрочастицами в анизотропной плазме” опубликована российскими учеными в журнале Scientific Reports в соавторстве с американскими физиками из астрофизического центра CASPER Бейлорского университета.

Силы, законы Ньютона — базовый урок по физике

А вот в неинерциальных системах скорость тела может меняться без силы.

Например: представьте, что вы стоите в центре автобуса, равномерно едущего по ровной дороге. Находясь внутри, вы даже не чувствуете, что автобус едет. В какой-то момент автобус резко тормозит и вас «бросает» вперед, хотя не действует никакая сила. То есть вы начинаете двигаться относительно автобуса без всякой причины. В таком случае автобус — это пример неинерциальной системы отсчета.

Неинерциальные системы отсчета — это системы, которые двигаются с ускорением. В таких системах вводятся так называемые силы инерции, чтобы при расчетах также можно было пользоваться законами Ньютона.

Нашу Землю можно условно отнести к инерциальным системам отсчета, поскольку вращение Земли есть ни что иное, как движение с центростремительным ускорением. Но так как Земля вращается достаточно медленно, то и центростремительное ускорение получается небольшим.

С высокой степенью точности инерциальной является гелиоцентрическая система отсчета (или система Коперника), начало которой помещено в центр Солнца, а оси направлены на далекие звезды. Вообще всякая система отсчета, движущаяся относительно какой-либо инерциальной системы поступательно, равномерно и прямолинейно, также является инерциальной. Например, поезд, идущий с постоянной скоростью по прямому участку пути.

Первый закон постулирует существование инерциальных систем отсчета, но не говорит, какую из множества таких систем предпочтительней выбирать. {n}Fi$.

Первый закон Ньютона: если на тело не действуют другие тела, то тело движется прямолинейно и равномерно: $\overrightarrow{F} = 0$.

Важно! Если есть ИСО, то любая другая система, движущаяся относительно неё прямолинейно и равномерно, также является инерциальной.

Второй закон Ньютон: ускорение тела прямо пропорционально силе, действующей на него, и обратно пропорционально его массе: $\overrightarrow{a} = \frac{\overrightarrow{F}}{m}$.

Другая запись формулы второго закона Ньютона (основное уравнение динамики): $\overrightarrow{F} = m \overrightarrow{a}$ .

Третий закон Ньютона: тела действуют друг на друга с силами, равными по модулю и противоположными по направлению: $\overrightarrow{F}_{12} = -\overrightarrow{F}_{21}$.

Второй закон Ньютона для системы тел: приращение импульса $\Delta \overrightarrow{P}$ системы тел равно по величине и по направлению импульсу внешних сил, действующих на тело, за то же время: $\Delta \overrightarrow{p} = \overrightarrow{F} \Delta t$.

Границы применимости: справедливы для материальных точек или поступательно движущихся тел; для скоростей много меньше скорости света в вакууме; выполняются в ИСО.

Решение задачи на применение второго закона Ньютона

Взаимодействие тел, третий закон Ньютона — ЗФТШ, МФТИ

Из анализов многочисленных опытов, как уже отмечалось, было получено соотношение масс взаимодействующих тел и их ускорений:

\[\frac{m_2}{m_1} = \frac{a_1}{a_2},\quad или \quad m_1a_1 = m_2a_2.\]

Но мы знаем из опытов, что при взаимодействии всегда ускорения тел противоположны друг другу: a→1↑↓a→2\vec a_1 \uparrow \downarrow \vec a_2, следовательно, m1a→1=-m2a→2m_1\vec a_1 = – m_2 \vec a_2.

Но произведение массы тела на ускорение этого тела равна действующей на это тело силе. Тогда 

F→1=-F→2\boxed{\vec F_1 = -\vec F_2}.

Данное утверждение и представляет собой третий закон Ньютона.

Третий закон Ньютона: При взаимодействии тела действуют друг на друга с силами, равными по величине, противоположными по направлению, одинаковыми по природе и лежащими на прямой, проходящей через центры тел.

Данные проявления встречаются всюду:

1) при столкновении (упругом или неупругом) тела деформируются, при этом появляются силы упругости. Первое тело действует на второе с силой F21F_{21}, а второе на первое с силой F12F_{12}. Причём обе силы по природе своей являются силами упругости – силами взаимодействия между молекулами (электромагнитными). Силы лежат на одной прямой, лежащей на линии точек приложения сил. Силы противоположны.

2) при гравитационном взаимодействии двух тел (Земля и Луна, или Солнце и Юпитер и т. д.) возникают две гравитационные силы, которые тоже противоположны и равны друг другу.

3) при взаимодействии прямоугольного тела, стоящего на поверхности стола, то же возникают две силы упругости: сила возникает потому, что стол деформировался (прогнулся, деформация изгиба см. далее), а сила возникает потому, что прямоугольное тело тоже деформировалось (сжалось под действием силы тяжести, подробнее см. далее). Обе силы равны друг другу и противоположны.

Рассмотрение примеров позволяет сформулировать следующие свойства сил, возникающих при взаимодействии:

силы всегда появляются (или исчезают) парами;

силы не компенсируют друг друга, т. к. приложены к разным телам;

силы одинаковой природы.

Пример 3. Для растяжения пружины жёсткостью 50 Н/м50\ \mathrm{Н}/\mathrm{м}, закреплённой одним концом на стене, на 20 см20\ \mathrm{см} требуется сила 10 Н10\ \mathrm{Н}. Какую силу нужно приложить к этой пружине, чтобы растянуть её на 20 см20\ \mathrm{см}, прикладывая силу с двух сторон и действуя в противоположных направлениях?

Решение.  В первом случае в растянутом состоянии пружина находилась в состоянии покоя. Следовательно, по второму закону Ньютона сила, приложенная к пружине со стороны руки, скомпенсирована силой, приложенной к пружине со стороны стены. Значит, стена действует на пружину с силой 10 Н10\ \mathrm{Н}. а) Первая пара сил: точка приложения силы со стороны руки неподвижна и находится в пружине, а сила упругости пружины приложена к точке, находящейся в руке, и тоже неподвижна. Эти две силы равны и противоположны по третьему закону Ньютона. б) Вторая пара сил: во второй паре взаимодействующих тел (стены и пружины) силы тоже равны и противоположны по тому же закону.

Во втором случае пружина тоже находится в покое. Только теперь одна из сил создаётся одной рукой, а вторая сила второй рукой. Сила, создаваемая стеной в первом случае, заменяется силой, создаваемой второй рукой, во втором. Понятно, что неподвижной пружина останется во втором случае только тогда, когда величина силы тоже сохранит первоначальное значение. Следовательно, во втором случае к пружине нужно приложить силу 10 Н10\ \mathrm{Н} с обеих сторон.

законов движения Ньютона | Определение, примеры и история

Законы движения Ньютона , отношения между силами, действующими на тело, и движением тела, впервые сформулированные английским физиком и математиком сэром Исааком Ньютоном.

Ньютон, Исаак; законы движения

Титульный лист книги Исаака Ньютона Philosophiae Naturalis Principia Mathematica (1687; Mathematical Principles of Natural Philosophy ), работы, в которой физик представил свои три закона движения.

Photos.com/Thinkstock

Популярные вопросы

Каковы законы движения Ньютона?

Законы движения Ньютона связывают движение объекта с силами, действующими на него. В первом законе объект не изменит своего движения, если на него не действует сила. Во втором законе сила, действующая на объект, равна его массе, умноженной на его ускорение. В третьем законе, когда два объекта взаимодействуют, они прикладывают друг к другу силы равной величины и противоположного направления.

Почему законы движения Ньютона важны?

Законы движения Ньютона важны, потому что они лежат в основе классической механики, одного из основных разделов физики.Механика – это изучение того, как объекты движутся или не двигаются, когда на них действуют силы.

Первый закон Ньютона гласит, что если тело находится в состоянии покоя или движется с постоянной скоростью по прямой линии, оно будет оставаться в покое или продолжать движение по прямой с постоянной скоростью, если на него не действует сила. Этот постулат известен как закон инерции. Закон инерции был впервые сформулирован Галилео Галилеем для горизонтального движения на Земле, а затем был обобщен Рене Декартом. До Галилея считалось, что любое горизонтальное движение требует прямой причины, но Галилей вывел из своих экспериментов, что движущееся тело будет оставаться в движении, если сила (например, трение) не заставит его остановиться.

баскетбол; Законы движения Ньютона

Когда баскетболист делает бросок в прыжке, мяч всегда следует по дуге. Мяч следует по этому пути, потому что его движение подчиняется законам движения сэра Исаака Ньютона.

© Mark Herreid / Shutterstock.com

Второй закон Ньютона – это количественное описание изменений, которые сила может вызвать при движении тела. Он утверждает, что скорость изменения импульса тела по величине и направлению равна силе, приложенной к нему.Импульс тела равен произведению его массы на скорость. Импульс, как и скорость, является векторной величиной, имеющей как величину, так и направление. Сила, приложенная к телу, может изменить величину импульса или его направление, или и то, и другое. Второй закон Ньютона – один из самых важных во всей физике. Для тела, масса которого м постоянна, это можно записать в виде F = м a , где F (сила) и a (ускорение) являются векторными величинами.Если на тело действует чистая сила, оно ускоряется в соответствии с уравнением. И наоборот, если тело не ускоряется, на него не действует действующая сила.

Третий закон Ньютона гласит, что при взаимодействии двух тел они прикладывают друг к другу силы, равные по величине и противоположные по направлению. Третий закон также известен как закон действия и противодействия. Этот закон важен при анализе задач статического равновесия, где все силы уравновешены, но он также применим к телам, движущимся равномерно или ускоренно.Силы, которые он описывает, являются реальными, а не просто бухгалтерскими приборами. Например, книга, лежащая на столе, прикладывает направленную вниз силу, равную ее весу на столе. Согласно третьему закону, стол прикладывает к книге равную и противоположную силу. Эта сила возникает из-за того, что вес книги заставляет стол слегка деформироваться, так что он толкает книгу назад, как спиральная пружина.

Получите подписку Britannica Premium и получите доступ к эксклюзивному контенту. Подпишитесь сейчас Законы

Ньютона впервые появились в его шедевре Philosophiae Naturalis Principia Mathematica (1687), широко известном как Principia .В 1543 году Николай Коперник предположил, что в центре Вселенной может находиться Солнце, а не Земля. За прошедшие годы Галилей, Иоганн Кеплер и Декарт заложили основы новой науки, которая заменит аристотелевское мировоззрение, унаследованное от древних греков, и объяснит работу гелиоцентрической вселенной. В Принципах Ньютон создал эту новую науку. Он разработал свои три закона, чтобы объяснить, почему орбиты планет представляют собой эллипсы, а не круги, что ему удалось, но оказалось, что он объяснил гораздо больше. Серия событий от Коперника до Ньютона известна под общим названием Научная революция.

В 20 веке законы Ньютона были заменены квантовой механикой и теорией относительности как наиболее фундаментальными законами физики. Тем не менее, законы Ньютона продолжают давать точное описание природы, за исключением очень маленьких тел, таких как электроны, или тел, движущихся со скоростью, близкой к скорости света. Квантовая механика и теория относительности сводятся к законам Ньютона для более крупных тел или для тел, движущихся медленнее.

Законы движения Ньютона | Исследовательский центр Гленна

Каковы законы движения Ньютона?

  1. Покоящийся объект остается неподвижным, а объект в движении остается в движении с постоянной скоростью и по прямой, если на него не действует неуравновешенная сила.
  2. Ускорение объекта зависит от массы объекта и величины приложенной силы.
  3. Каждый раз, когда один объект оказывает силу на другой объект, второй объект оказывает равное и противоположное воздействие на первый.

Сэр Исаак Ньютон работал во многих областях математики и физики. Он разработал теории гравитации в 1666 году, когда ему было всего 23 года. В 1686 году он представил свои три закона движения в «Principia Mathematica Philosophiae Naturalis».

Разработав три закона движения, Ньютон произвел революцию в науке. Законы Ньютона вместе с законами Кеплера объяснили, почему планеты движутся по эллиптическим орбитам, а не по кругу.

Ниже представлен небольшой фильм с участием Орвилла и Уилбура Райтов и обсуждение того, как законы движения Ньютона применимы к полету их самолета.

Первый закон Ньютона: инерция

Покоящийся объект остается неподвижным, а объект в движении остается в движении с постоянной скоростью и по прямой, если на него не действует неуравновешенная сила.

Первый закон Ньютона гласит, что каждый объект будет оставаться в покое или в равномерном движении по прямой линии, если только он не будет вынужден изменить свое состояние под действием внешней силы. Эта тенденция сопротивляться изменениям в состоянии движения составляет инерции .Нет чистой силы , действующей на объект (если все внешние силы нейтрализуют друг друга). Тогда объект будет поддерживать постоянную скорость . Если эта скорость равна нулю, объект остается в покое. Если на объект действует внешняя сила, скорость изменится из-за силы.

Примеры инерции, связанные с аэродинамикой:

  • Движение самолета, когда пилот меняет положение дроссельной заслонки двигателя.
  • Движение шара, падающего в атмосфере.
  • Модель ракеты, запускаемой в атмосферу.
  • Движение воздушного змея при перемене ветра.

Второй закон Ньютона: сила

Ускорение объекта зависит от массы объекта и количества приложенной силы.

Его второй закон определяет, что сила равна изменению импульса (масса, умноженная на скорость) за изменение во времени. Импульс определяется как масса объекта м , умноженная на его скорость V .

Предположим, что у нас есть самолет в точке «0», определяемой его местоположением X0 и временем t0 . Самолет имеет массу m0 и движется со скоростью V0 . Внешняя сила F , приложенная к самолету, показанному выше, перемещает его в точку «1». Новое местоположение самолета X1 , время t1 .

Масса и скорость самолета изменяются во время полета до значений м1 и V1 .Второй закон Ньютона может помочь нам определить новые значения V1 и m1 , если мы знаем, насколько велика сила F . Давайте просто возьмем разницу между условиями в точке «1» и условиями в точке «0».

F = (m1 * V1 – m0 * V0) / (t1 – t0)

Второй закон Ньютона говорит об изменениях количества движения (м * В), поэтому на данный момент мы не можем отделить, насколько изменилась масса и насколько изменилась скорость. Мы знаем только, насколько изменился продукт (m * V).

Предположим, что масса остается постоянной величиной, равной м . Это предположение довольно хорошо для самолета, единственное изменение массы будет для топлива, сожженного между точкой «1» и точкой «0». Вес топлива, вероятно, невелик по сравнению с весом остальной части самолета, особенно если мы будем смотреть только на небольшие изменения во времени. Если бы мы обсуждали полет бейсбольного мяча, то, конечно, масса остается постоянной. Но если бы мы обсуждали полет баллонной ракеты, то масса не остается постоянной, и мы можем смотреть только на изменение количества движения.Для постоянной массы м второй закон Ньютона выглядит так:

F = m * (V1 – V0) / (t1 – t0)

Изменение скорости, деленное на изменение во времени, и есть определение ускорения a . Затем второй закон сводится к более знакомому произведению массы и ускорения:

F = м * а

Помните, что это соотношение подходит только для объектов с постоянной массой. Это уравнение говорит нам, что объект, на который действует внешняя сила, будет ускоряться и что величина ускорения пропорциональна величине силы.Величина ускорения также обратно пропорциональна массе объекта; при равных силах более тяжелый объект будет испытывать меньшее ускорение, чем более легкий объект. Рассматривая уравнение количества движения, сила вызывает изменение скорости; и аналогично изменение скорости порождает силу. Уравнение работает в обоих направлениях.

Скорость, сила, ускорение и импульс имеют как величину , так и направление , связанное с ними. Ученые и математики называют это векторной величиной.Показанные здесь уравнения на самом деле являются векторными уравнениями и могут применяться в каждом из направлений компонентов. Мы рассмотрели только одно направление, и, как правило, объект движется во всех трех направлениях (вверх-вниз, влево-вправо, вперед-назад).

Пример силы, связанной с аэродинамикой:

Третий закон Ньютона: действие и противодействие

Каждый раз, когда один объект оказывает силу на второй объект, второй объект оказывает на первый равную и противоположную силу.

Его третий закон гласит, что на каждые действия (силы) в природе существует равное и противоположное противодействие .Если объект A оказывает силу на объект B, объект B также оказывает равную и противоположную силу на объект A. Другими словами, силы возникают в результате взаимодействий.

Примеры действий и противодействий, связанных с аэродинамикой:

  • Движение подъемной силы от аэродинамического профиля, воздух отклоняется вниз под действием аэродинамического профиля, и в ответ крыло толкается вверх.
  • Движение вращающегося шара, воздух отклоняется в одну сторону, а мяч реагирует движением в противоположную сторону.
  • Движение реактивного двигателя вызывает тягу, и горячие выхлопные газы выходят из задней части двигателя, а сила тяги создается в противоположном направлении.

Обзор законов движения Ньютона
1. Первый закон движения Ньютона (инерция) Покоящийся объект остается неподвижным, а объект в движении остается в движении с постоянной скоростью и по прямой, если на него не действует неуравновешенная сила.
2. Второй закон движения Ньютона (Сила) Ускорение объекта зависит от массы объекта и величины приложенной силы.
3. Третий закон движения Ньютона (Действие и противодействие) Каждый раз, когда один объект оказывает силу на другой объект, второй объект оказывает равное и противоположное воздействие на первый.

Второй закон движения Ньютона

Первый закон движения Ньютона предсказывает поведение объектов, для которых все существующие силы уравновешены. Первый закон – иногда называемый законом инерции – гласит, что если силы, действующие на объект, уравновешены, то ускорение этого объекта будет 0 м / с / с.Объекты в равновесии (состояние, при котором все силы уравновешены) не будут ускоряться. Согласно Ньютону, объект будет ускоряться только в том случае, если на него действует чистая или неуравновешенная сила. Присутствие неуравновешенной силы ускоряет объект, изменяя его скорость, направление или одновременно скорость и направление.


Второй закон движения Ньютона относится к поведению объектов, для которых все существующие силы не сбалансированы.Второй закон гласит, что ускорение объекта зависит от двух переменных – чистой силы, действующей на объект, и массы объекта. Ускорение объекта напрямую зависит от чистой силы, действующей на объект, и обратно – от массы объекта. По мере увеличения силы, действующей на объект, ускорение объекта увеличивается. По мере увеличения массы объекта ускорение объекта уменьшается.

БОЛЬШОЕ Уравнение

Второй закон движения Ньютона можно формально сформулировать следующим образом:

Ускорение объекта, создаваемое чистой силой, прямо пропорционально величине чистой силы в том же направлении, что и чистая сила, и обратно пропорционально массе объекта.

Это словесное утверждение можно выразить в виде уравнения следующим образом:

a = F нетто / м

Приведенное выше уравнение часто преобразовывается в более знакомую форму, как показано ниже. Чистая сила равна произведению массы на ускорение.

F net = m a

Во всем этом обсуждении упор был сделан на чистую силу .Ускорение прямо пропорционально чистой силе ; чистая сила равна массе, умноженной на ускорение; ускорение в том же направлении, что и чистая сила ; ускорение создается чистой силой . СЕТЕВАЯ СИЛА. Важно помнить об этом различии. Не используйте в приведенном выше уравнении значение просто «какой-либо старой силы». Это чистая сила, связанная с ускорением. Как обсуждалось в предыдущем уроке, результирующая сила – это векторная сумма всех сил.Если известны все индивидуальные силы, действующие на объект, то можно определить результирующую силу. При необходимости просмотрите этот принцип, вернувшись к практическим вопросам в Уроке 2.

В соответствии с приведенным выше уравнением единица силы равна единице массы, умноженной на единицу ускорения. Подставив стандартные метрические единицы для силы, массы и ускорения в приведенное выше уравнение, можно записать следующую эквивалентность единиц.

1 Ньютон = 1 кг • м / с 2

Определение стандартной метрической единицы силы определяется приведенным выше уравнением.Один Ньютон определяется как количество силы, необходимое для придания 1 кг массы ускорения 1 м / с / с.

Ваша очередь практиковаться

Сеть F = m • уравнение часто используется при решении алгебраических задач. Приведенную ниже таблицу можно заполнить, подставив в уравнение и решив неизвестную величину. Попробуйте сами, а затем используйте кнопки, чтобы просмотреть ответы.

Чистая сила (н.) Масса (кг) Разгон (м / с / с)
1. 10 2
2. 20 2
3. 20 4
4. 2 5
5. 10 10


Второй закон Ньютона как руководство к мышлению

Числовая информация в таблице выше демонстрирует некоторые важные качественные отношения между силой, массой и ускорением.Сравнивая значения в строках 1 и 2, можно увидеть, что удвоение чистой силы приводит к удвоению ускорения (если масса остается постоянной). Точно так же сравнение значений в строках 2 и 4 показывает, что уменьшение вдвое чистой силы на приводит к уменьшению вдвое ускорения (если масса остается постоянной). Ускорение прямо пропорционально чистой силе.

Кроме того, качественную взаимосвязь между массой и ускорением можно увидеть, сравнив числовые значения в приведенной выше таблице.Обратите внимание на строки 2 и 3, что удвоение массы приводит к уменьшению вдвое ускорения (если сила остается постоянной). Аналогичным образом, строки 4 и 5 показывают, что уменьшение вдвое массы на приводит к удвоению ускорения (если сила остается постоянной). Ускорение обратно пропорционально массе.

Анализ табличных данных показывает, что такое уравнение, как F net = m * a, может помочь понять, как изменение одной величины может повлиять на другую величину.Какое бы изменение ни производилось в чистой силе, такое же изменение произойдет и с ускорением. Удвойте, утроите или учетверите чистую силу, и ускорение будет делать то же самое. С другой стороны, какое бы изменение массы ни производилось, с ускорением будет происходить противоположное или обратное изменение. Удвойте, утроите или учетверите массу, и ускорение составит половину, одну треть или одну четвертую от первоначального значения.


Направление чистой силы и ускорения

Как указано выше, направление результирующей силы совпадает с направлением ускорения.Таким образом, если известно направление ускорения, то известно и направление результирующей силы. Рассмотрим две диаграммы падения масла ниже для ускорения автомобиля. По диаграмме определите направление чистой силы, действующей на автомобиль. Затем нажмите кнопки, чтобы просмотреть ответы. (При необходимости проверьте ускорение предыдущего блока.)

В заключение, второй закон Ньютона дает объяснение поведения объектов, на которых силы не уравновешиваются.Закон гласит, что несбалансированные силы заставляют объекты ускоряться с ускорением, которое прямо пропорционально чистой силе и обратно пропорционально массе.


Мы хотели бы предложить … Иногда просто прочитать об этом недостаточно. Вы должны взаимодействовать с ним! И это именно то, что вы делаете, когда используете один из интерактивных материалов The Physics Classroom. Мы хотели бы предложить вам совместить чтение этой страницы с использованием нашего Force Interactive.Вы можете найти его в разделе Physics Interactives на нашем сайте. Force Interactive позволяет учащемуся исследовать влияние изменений приложенной силы, чистой силы, массы и трения на ускорение объекта.


Ракетостроение!

Ракеты НАСА (и другие) ускоряются от стартовой площадки, сжигая огромное количество топлива. Когда топливо сгорает и расходуется для приведения в движение ракеты, масса ракеты изменяется.Таким образом, одна и та же движущая сила может привести к увеличению значений ускорения с течением времени. Используйте виджет Rocket Science ниже, чтобы изучить этот эффект.

Проверьте свое понимание

1. Определите ускорения, возникающие при приложении чистой силы 12 Н к объекту массой 3 кг, а затем к объекту массой 6 кг.


2. К энциклопедии прилагается чистая сила 15 Н, которая заставляет ее ускоряться со скоростью 5 м / с 2 .Определите массу энциклопедии.


3. Предположим, что салазки ускоряются со скоростью 2 м / с 2 . Если чистая сила утроится, а масса – вдвое, то каково новое ускорение салазок?


4. Предположим, что салазки ускоряются со скоростью 2 м / с 2 . Если чистая сила утроится, а масса уменьшится вдвое, то каково новое ускорение салазок?

Законы движения Ньютона | Живая наука

Три закона движения сэра Исаака Ньютона описывают движение массивных тел и их взаимодействие.Хотя сегодня законы Ньютона могут показаться нам очевидными, более трех веков назад они считались революционными.

Ньютон был одним из самых влиятельных ученых всех времен. Его идеи легли в основу современной физики. Он опирался на идеи, выдвинутые из работ предыдущих ученых, включая Галилея и Аристотеля, и смог доказать некоторые идеи, которые в прошлом были только теориями. Он изучал оптику, астрономию и математику – он изобрел исчисление. (Немецкому математику Готфриду Лейбницу также приписывают независимое развитие примерно в то же время.)

Ньютон, пожалуй, наиболее известен своими работами по изучению гравитации и движения планет. По настоянию астронома Эдмона Галлея после признания того, что он потерял свое доказательство эллиптических орбит за несколько лет до этого, Ньютон опубликовал свои законы в 1687 году в своей основополагающей работе «Philosophiæ Naturalis Principia Mathematica» («Математические принципы естественной философии»), в которой он формализовал описание того, как массивные тела движутся под действием внешних сил.

Формулируя свои три закона, Ньютон упростил рассмотрение массивных тел, рассматривая их как математические точки без размера и вращения.Это позволило ему игнорировать такие факторы, как трение, сопротивление воздуха, температуру, свойства материала и т. Д., И сосредоточиться на явлениях, которые можно описать исключительно в терминах массы, длины и времени. Следовательно, эти три закона нельзя использовать для точного описания поведения больших твердых или деформируемых объектов; однако во многих случаях они обеспечивают достаточно точные приближения.

Законы Ньютона относятся к движению массивных тел в инерциальной системе отсчета , иногда называемой ньютоновской системой отсчета , хотя сам Ньютон никогда не описывал такую ​​систему отсчета.Инерциальная система отсчета может быть описана как трехмерная система координат, которая либо неподвижна, либо находится в равномерном линейном движении, то есть не ускоряется и не вращается. Он обнаружил, что движение в такой инерциальной системе отсчета можно описать тремя простыми законами.

Первый закон движения гласит: «Покоящееся тело будет оставаться в покое, а тело в движении будет оставаться в движении, если на него не действует внешняя сила». Это просто означает, что вещи не могут запускаться, останавливаться или менять направление сами по себе.Требуется некоторая сила, действующая на них извне, чтобы вызвать такое изменение. Это свойство массивных тел сопротивляться изменениям в их состоянии движения иногда называют инерцией .

Второй закон движения описывает, что происходит с массивным телом, когда на него действует внешняя сила. В нем говорится: «Сила, действующая на объект, равна массе этого объекта, умноженной на его ускорение». Это записывается в математической форме как F = m a , где F – сила, m – масса и a – ускорение.Жирными буквами обозначено, что сила и ускорение векторных величин, что означает, что они имеют как величину, так и направление. Сила может быть одной силой или векторной суммой более чем одной силы, которая представляет собой результирующую силу после объединения всех сил.

Когда на массивное тело действует постоянная сила, она заставляет его ускоряться, то есть изменять свою скорость с постоянной скоростью. В простейшем случае сила, приложенная к неподвижному объекту, заставляет его ускоряться в направлении силы.Однако, если объект уже находится в движении или если эта ситуация рассматривается из движущейся системы отсчета, это тело может казаться ускоряющимся, замедляющимся или меняющим направление в зависимости от направления силы и направлений, в которых объект и системы отсчета движутся относительно друг друга.

Третий закон движения гласит: «На каждое действие есть равное и противоположное противодействие». Этот закон описывает, что происходит с телом, когда оно воздействует на другое тело. Силы всегда возникают парами, поэтому, когда одно тело толкает другое, второе тело с такой же силой отталкивается.Например, когда вы толкаете тележку, тележка толкает вас назад; когда вы тянете за веревку, веревка тянется к вам; когда сила тяжести прижимает вас к земле, земля толкает вас вверх по ногам; и когда ракета воспламеняет свое топливо позади себя, расширяющийся выхлопной газ толкает ракету, заставляя ее ускоряться.

Если один объект намного, намного массивнее другого, особенно в случае, когда первый объект привязан к Земле, практически все ускорение передается второму объекту, и ускорение первого объекта может быть безопасно игнорировать.Например, если бы вы бросили бейсбольный мяч на запад, вам не нужно было бы учитывать, что вы фактически заставили вращение Земли хоть сколько-нибудь ускоряться, пока мяч находился в воздухе. Однако, если бы вы стояли на роликовых коньках и бросили шар для боулинга вперед, вы бы начали двигаться назад с заметной скоростью.

Три закона были подтверждены бесчисленными экспериментами за последние три столетия, и они все еще широко используются по сей день для описания типов объектов и скоростей, с которыми мы сталкиваемся в повседневной жизни.Они составляют основу того, что сейчас известно как классической механики , которая представляет собой исследование массивных объектов, которые больше, чем очень маленькие масштабы, рассматриваемые квантовой механикой, и движутся медленнее, чем очень высокие скорости, рассматриваемые релятивистской механикой.

Дополнительные ресурсы

Второй закон движения Ньютона: концепция системы

Цели обучения

К концу этого раздела вы сможете:

  • Определите чистую силу, внешнюю силу и систему.
  • Поймите второй закон движения Ньютона.
  • Примените второй закон Ньютона для определения веса объекта.

Второй закон движения Ньютона тесно связан с первым законом движения Ньютона. Он математически устанавливает причинно-следственную связь между силой и изменениями в движении. Второй закон движения Ньютона является более количественным и широко используется для расчета того, что происходит в ситуациях, связанных с действием силы. Прежде чем мы сможем записать второй закон Ньютона в виде простого уравнения, дающего точное соотношение силы, массы и ускорения, нам нужно отточить некоторые идеи, которые уже упоминались.

Во-первых, что мы подразумеваем под изменением движения? Ответ заключается в том, что изменение движения эквивалентно изменению скорости. Изменение скорости по определению означает наличие ускорения . Первый закон Ньютона гласит, что чистая внешняя сила вызывает изменение движения; таким образом, мы видим, что чистая внешняя сила вызывает ускорение .

Сразу возникает еще один вопрос. Что мы подразумеваем под внешней силой? Интуитивное представление о внешнем является правильным – внешняя сила действует извне из интересующей системы .Например, на рис. 1 (а) интересующая система – это вагон плюс ребенок в нем. Две силы, действующие со стороны других детей, – это внешние силы. Между элементами системы действует внутренняя сила. Снова посмотрев на рисунок 1 (а), сила, которую ребенок в повозке прикладывает, чтобы повиснуть на ней, является внутренней силой между элементами интересующей системы. Согласно первому закону Ньютона, только внешние силы влияют на движение системы. (Внутренние силы фактически отменяются, как мы увидим в следующем разделе.) Вы должны определить границы системы, прежде чем сможете определить, какие силы являются внешними . Иногда система очевидна, тогда как в других случаях определение границ системы более тонкое. Концепция системы является фундаментальной для многих областей физики, как и правильное применение законов Ньютона. Эта концепция будет многократно пересмотрена на нашем пути через физику.

Рис. 1. Различные силы, действующие на одну и ту же массу, вызывают разное ускорение.а) Двое детей толкают тележку с ребенком в ней. Показаны стрелки, представляющие все внешние силы. Интересующая система – это повозка и ее водитель. Вес системы w и опора земли N также показаны для полноты и, как предполагается, отменяют. Вектор f представляет трение, действующее на вагон, и действует влево, противодействуя движению вагона. (b) Все внешние силы, действующие на систему, в сумме образуют чистую силу, F net .На диаграмме свободного тела показаны все силы, действующие на интересующую систему. Точка представляет собой центр масс системы. Каждый вектор силы простирается от этой точки. Поскольку справа действуют две силы, мы рисуем векторы коллинеарно. (c) Большая чистая внешняя сила вызывает большее ускорение ( a ‘> a ), когда взрослый толкает ребенка.

Теперь кажется разумным, что ускорение должно быть прямо пропорционально и в том же направлении, что и чистая (полная) внешняя сила, действующая на систему.Это предположение было проверено экспериментально и показано на рисунке 1. В части (а) меньшая сила вызывает меньшее ускорение, чем большая сила, показанная в части (с). Для полноты картины также показаны вертикальные силы; предполагается, что они отменяются, поскольку нет ускорения в вертикальном направлении. Вертикальные силы – это вес w и опора на землю N , а горизонтальная сила f представляет собой силу трения. Они будут обсуждаться более подробно в следующих разделах.На данный момент мы определим трение как силу, которая противодействует движению друг мимо друга соприкасающихся объектов. На рисунке 1 (b) показано, как векторы, представляющие внешние силы, складываются, чтобы получить результирующую силу,

.

Ф нетто .

Чтобы получить уравнение для второго закона Ньютона, сначала запишем соотношение ускорения и чистой внешней силы как пропорциональность

[латекс] \ text {a} \ propto {\ text {F} _ {net}} \\ [/ latex]

, где символ ∝ означает «пропорционально», а F net – это чистая внешняя сила .(Чистая внешняя сила представляет собой векторную сумму всех внешних сил и может быть определена графически, используя метод “голова к хвосту”, или аналитически, используя компоненты. Методы такие же, как и для сложения других векторов, и рассматриваются в двумерной кинематике.) Эта пропорциональность выражает то, что мы сказали словами: ускорение прямо пропорционально чистой внешней силе . После выбора интересующей системы важно определить внешние силы и игнорировать внутренние.Это огромное упрощение – не учитывать многочисленные внутренние силы, действующие между объектами внутри системы, такие как мышечные силы в теле ребенка, не говоря уже о мириадах сил между атомами в объектах, но, делая это, мы можем легко решить некоторые очень сложные проблемы с минимальной ошибкой благодаря нашему упрощению.

Теперь также кажется разумным, что ускорение должно быть обратно пропорционально массе системы. Другими словами, чем больше масса (инерция), тем меньше ускорение, создаваемое данной силой.И действительно, как показано на рисунке 2, та же самая чистая внешняя сила, приложенная к автомобилю, вызывает гораздо меньшее ускорение, чем при приложении к баскетбольному мячу. Пропорциональность записывается как

.

[латекс] \ text {a} \ propto {\ frac {1} {m}} \\ [/ latex]

, где м – масса системы. Эксперименты показали, что ускорение прямо обратно пропорционально массе, точно так же, как оно прямо линейно пропорционально чистой внешней силе.

Рис. 2. Одна и та же сила, действующая на системы разной массы, вызывает разное ускорение.(a) Баскетболист толкает баскетбольный мяч, чтобы сделать передачу. (Влияние силы тяжести на мяч игнорируется.) (B) Тот же игрок оказывает идентичную силу на остановившийся внедорожник и производит гораздо меньшее ускорение (даже если трение незначительно). (c) Диаграммы свободного тела идентичны, что позволяет напрямую сравнить две ситуации. По мере того, как вы будете решать больше задач, появится серия паттернов для диаграммы свободного тела.

Было обнаружено, что ускорение объекта зависит только от чистой внешней силы и массы объекта.Объединение двух только что приведенных пропорциональностей дает второй закон движения Ньютона.

Второй закон движения Ньютона

Ускорение системы прямо пропорционально чистой внешней силе, действующей на систему, и в том же направлении, и обратно пропорционально ее массе. В форме уравнения второй закон движения Ньютона равен

.

[латекс] {\ text {a}} = \ frac {{{\ text {F}}} _ {\ text {net}}} {m} \\ [/ latex].

Это часто записывается в более знакомой форме

F нетто = м a .

Если рассматривать только величину силы и ускорения, это уравнение просто

F net = ma.

Хотя эти последние два уравнения на самом деле одинаковы, первое дает больше понимания того, что означает второй закон Ньютона. Закон – причинно-следственная связь между тремя величинами, которая не просто основана на их определениях. Справедливость второго закона полностью основана на экспериментальной проверке.

F net = м используется для определения единиц силы в терминах трех основных единиц массы, длины и времени. Единица силы в системе СИ называется ньютон (сокращенно Н) и представляет собой силу, необходимую для ускорения системы массой 1 кг со скоростью 1 м / с 2 . То есть, поскольку F net = м a ,

1 Н = 1 кг ⋅ м / с 2 .

В то время как почти весь мир использует ньютон в качестве единицы силы, в Соединенных Штатах наиболее известной единицей силы является фунт (фунт), где 1 N = 0.225 фунтов

Вес и гравитационная сила

Когда объект падает, он ускоряется к центру Земли. Второй закон Ньютона гласит, что общая сила, действующая на объект, отвечает за его ускорение. Если сопротивление воздуха незначительно, результирующая сила, действующая на падающий объект, – это сила тяжести, обычно называемая его весом Вт. Вес можно обозначить как вектор w, потому что он имеет направление; вниз – это, по определению, направление силы тяжести, и, следовательно, вес – это сила, направленная вниз.Величина веса обозначается как w . Галилей показал, что при отсутствии сопротивления воздуха все объекты падают с одинаковым ускорением g . Используя результат Галилея и второй закон Ньютона, мы можем вывести уравнение для веса.

Рассмотрим объект массой м , падающий на Землю. Он испытывает только направленную вниз силу тяжести, величина которой составляет w . Второй закон Ньютона гласит, что величина чистой внешней силы, действующей на объект, составляет F net = ma .Поскольку на объект действует только сила тяжести, направленная вниз, F net = w . Мы знаем, что ускорение объекта под действием силы тяжести составляет g , или a = g . Подставляя их во второй закон Ньютона, получаем

Масса

Это уравнение для веса – силы тяжести, действующей на массу m:

w = мг

Так как g = 9,80 м / с 2 на Земле, вес 1.0 кг объекта на Земле составляет 9,8 Н, как мы видим:

w = мг = (1,0 кг) (9,80 м / с 2 ) = 9,8 Н.

Напомним, что g может принимать положительное или отрицательное значение в зависимости от положительного направления в системе координат. Обязательно учтите это при решении задач с весом.

Когда чистая внешняя сила, действующая на объект, равна его весу, мы говорим, что оно находится в свободном падении . То есть единственная сила, действующая на объект, – это сила тяжести.В реальном мире, когда объекты падают вниз к Земле, они никогда не находятся в состоянии свободного падения, потому что на объект всегда действует некоторая восходящая сила из воздуха.

Ускорение свободного падения g незначительно меняется по поверхности Земли, поэтому вес объекта зависит от местоположения и не является внутренним свойством объекта. Вес резко меняется, если человек покидает поверхность Земли. На Луне, например, ускорение свободного падения всего 1.67 м / с 2 . Таким образом, масса в 1,0 кг имеет вес 9,8 Н на Земле и всего около 1,7 Н на Луне.

Самое широкое определение веса в этом смысле состоит в том, что вес объекта – это сила тяжести, действующая на него со стороны ближайшего большого тела , такого как Земля, Луна, Солнце и так далее. Это наиболее распространенное и полезное определение веса в физике. Однако оно кардинально отличается от определения веса, используемого НАСА и популярными СМИ в отношении космических путешествий и исследований.Когда они говорят о «невесомости» и «микрогравитации», они на самом деле имеют в виду явление, которое в физике мы называем «свободным падением». Мы будем использовать приведенное выше определение веса и проведем тщательное различие между свободным падением и фактической невесомостью.

Важно знать, что вес и масса – очень разные физические величины, хотя они тесно связаны. Масса – это количество материи (сколько «вещества») и не изменяется в классической физике, тогда как вес – это сила тяжести, которая зависит от силы тяжести.Заманчиво приравнять эти два понятия, поскольку большинство наших примеров имеет место на Земле, где вес объекта лишь немного зависит от его местоположения. Кроме того, в повседневном языке термины масса и масса используются взаимозаменяемо; например, в наших медицинских записях наш «вес» часто указывается в килограммах, но никогда в правильных единицах – ньютонах.

Распространенные заблуждения: масса против веса

В обиходе масса и вес часто используются как синонимы.Однако в науке эти термины существенно отличаются друг от друга. Масса – это мера количества вещества в объекте. Типичной мерой массы является килограмм (или «слизняк» в английских единицах измерения). С другой стороны, вес – это мера силы тяжести, действующей на объект. Вес равен массе объекта (м), умноженной на ускорение свободного падения (g). Как и любая другая сила, вес измеряется в ньютонах (или фунтах в английских единицах). Предполагая, что масса объекта остается неизменной, она останется неизменной независимо от его местоположения.Однако, поскольку вес зависит от ускорения свободного падения, вес объекта может измениться на , когда объект входит в область с большей или меньшей силой тяжести. Например, ускорение свободного падения на Луне составляет 1,67 м / с 2 (что намного меньше ускорения свободного падения на Земле, 9,80 м / с 2 ). Если вы измерили свой вес на Земле, а затем измерили свой вес на Луне, вы бы обнаружили, что «весите» гораздо меньше, хотя и не выглядите худее.Это потому, что сила тяжести на Луне слабее. Фактически, когда люди говорят, что они «худеют», они на самом деле имеют в виду, что они теряют «массу» (что, в свою очередь, заставляет их весить меньше).

Эксперимент на вынос: масса и вес

Что измеряют весы для ванной? Когда вы стоите на весах в ванной, что происходит с весами? Слегка угнетает. Весы содержат пружины, которые сжимаются пропорционально вашему весу – подобно резиновым лентам, расширяющимся при натяжении.Пружины позволяют измерить ваш вес (для объекта, который не ускоряется). Это сила в ньютонах (или фунтах). В большинстве стран результат делится на 9,80, чтобы получить значение в килограммах. Весы измеряют вес, но откалиброваны для предоставления информации о массе. Стоя на весах в ванной, нажмите на соседний стол. Что происходит с чтением? Почему? Будет ли ваша шкала измерять ту же «массу» на Земле, что и на Луне?

Пример 1.Какое ускорение может дать человек, толкая газонокосилку?

Предположим, что чистая внешняя сила (толчок минус трение), действующая на газонокосилку, составляет 51 Н (около 11 фунтов) параллельно земле. Масса косилки 24 кг. Какое у него ускорение?

Рис. 3. Сила полезного действия на газонокосилку справа составляет 51 Н. С какой скоростью газонокосилка ускоряется вправо?

Стратегия

Поскольку даны F net и m, ускорение можно рассчитать непосредственно из второго закона Ньютона, как указано в F net = м a .{2} \\ [/ латекс].

Обсуждение

Направление ускорения совпадает с направлением результирующей силы, параллельной земле. В этом примере нет информации об отдельных внешних силах, действующих на систему, но мы можем кое-что сказать об их относительных величинах. Например, сила, прикладываемая человеком, толкающим косилку, должна быть больше, чем трение, препятствующее движению (поскольку мы знаем, что косилка движется вперед), а вертикальные силы должны нейтрализоваться, если не должно быть ускорения в вертикальном направлении ( косилка движется только горизонтально).Обнаруженное ускорение достаточно мало, чтобы быть приемлемым для человека, толкающего косилку. Такое усилие не будет длиться слишком долго, потому что человек скоро достигнет максимальной скорости.

Пример 2. Какая ракетная тяга разгоняет салазки?

До пилотируемых космических полетов ракетные сани использовались для испытания самолетов, ракетного оборудования и физиологических воздействий на людей на высоких скоростях. Они состояли из платформы, которая была установлена ​​на одной или двух направляющих и приводилась в движение несколькими ракетами.Вычислите величину силы, прилагаемой каждой ракетой, которая называется ее тягой T, для четырехракетной двигательной установки, показанной на рисунке 4. Начальное ускорение салазок составляет 49 м / с2, масса системы – 2100 кг, а сила трения. противодействие движению, как известно, составляет 650 Н.

Рис. 4. Салазки испытывают ракетную тягу, которая ускоряет их вправо. Каждая ракета создает одинаковую тягу T. Как и в других ситуациях, когда есть только горизонтальное ускорение, вертикальные силы нейтрализуются.Земля оказывает на систему направленную вверх силу N, равную по величине и противоположную по направлению ее весу w. Система здесь – сани, его ракеты и всадник, поэтому никакие силы между этими объектами не рассматриваются. Стрелка, обозначающая трение (f), нарисована крупнее шкалы.

Стратегия

Несмотря на то, что существуют силы, действующие вертикально и горизонтально, мы предполагаем, что вертикальные силы компенсируются, поскольку нет вертикального ускорения. Это оставляет нам только горизонтальные силы и более простую одномерную задачу.Направления указываются знаками плюс или минус, при этом вправо принимается за положительное направление. См. Диаграмму свободного тела на рисунке.

Решение

Поскольку ускорение, масса и сила трения даны, мы начнем со второго закона Ньютона и ищем способы найти тягу двигателей. Поскольку мы определили направление силы и ускорения как действующие «вправо», нам нужно учитывать в расчетах только величины этих величин.Следовательно, мы начинаем с

F net = ma,

, где F net – чистая сила в горизонтальном направлении. Из рисунка 4 видно, что тяга двигателя увеличивается, а трение противодействует тяги. В форме уравнения чистая внешняя сила равна

.

Подставляя это во второй закон Ньютона, получаем

Используя небольшую алгебру, мы решаем общую тягу 4 T :

Подстановка известных значений дает

[латекс] 4T = \ text {ma} + f = \ left (\ text {2100 кг} \ right) \ left ({\ text {49 м / с}} ^ {2} \ right) + \ text { 650 Н} \ [/ латекс]

[латекс] 4Т = 1.{4} \ text {N} \\ [/ latex].

Обсуждение

Цифры довольно большие, поэтому результат может вас удивить. Подобные эксперименты проводились в начале 1960-х годов для проверки пределов человеческой выносливости и установки, предназначенной для защиты людей при аварийных выбросах реактивных истребителей. Была получена скорость 1000 км / ч с ускорением 45 g. (Напомним, что g, ускорение свободного падения, составляет 9,80 м / с 2 . Когда мы говорим, что ускорение составляет 45 g, это 45 × 9.80 м / с 2 , что составляет примерно 440 м / с 2 .) Хотя живые предметы больше не используются, с помощью ракетных саней была получена сухопутная скорость 10 000 км / ч. В этом примере, как и в предыдущем, интересующая система очевидна. В последующих примерах мы увидим, что выбор интересующей системы имеет решающее значение, и этот выбор не всегда очевиден.

Второй закон движения Ньютона – это больше, чем определение; это соотношение между ускорением, силой и массой.Это может помочь нам делать прогнозы. Каждую из этих физических величин можно определить независимо, поэтому второй закон говорит нам что-то основное и универсальное о природе. В следующем разделе представлен третий и последний закон движения.

Сводка раздела

  • Ускорение, a , определяется как изменение скорости, означающее изменение ее величины или направления, или и того, и другого.
  • Внешняя сила – это сила, действующая на систему извне, в отличие от внутренних сил, которые действуют между компонентами внутри системы.
  • Второй закон движения Ньютона гласит, что ускорение системы прямо пропорционально и в том же направлении, что и чистая внешняя сила, действующая на систему, и обратно пропорционально ее массе.
  • В форме уравнения второй закон движения Ньютона выглядит так: [latex] {\ text {a}} = \ frac {{{\ text {F}}} _ {\ text {net}}} {m} \\ [/ латекс].
  • Это часто записывается в более привычной форме: F net = m a .
  • Вес w объекта определяется как сила тяжести, действующая на объект массой m .Объект испытывает ускорение свободного падения g :

    w = m g .

  • Если единственная сила, действующая на объект, вызвана гравитацией, объект находится в свободном падении.
  • Трение – это сила, которая препятствует движению соприкасающихся объектов друг за другом.

Концептуальные вопросы

1. Какое утверждение верно? (а) Чистая сила вызывает движение. (b) Чистая сила вызывает изменение движения. Объясните свой ответ и приведите пример.

2. Почему мы можем пренебрегать такими силами, как силы, удерживающие тело вместе, когда мы применяем второй закон движения Ньютона?

3. Объясните, как выбор «интересующей системы» влияет на то, какие силы необходимо учитывать при применении второго закона движения Ньютона.

4. Опишите ситуацию, в которой чистая внешняя сила, действующая на систему, не равна нулю, но ее скорость остается постоянной.

5. Система может иметь ненулевую скорость, в то время как чистая внешняя сила, действующая на нее , равна нулю.Опишите такую ​​ситуацию.

6. Камень брошен вверх. Какая чистая внешняя сила действует на скалу, когда она находится на вершине своей траектории?

7. (a) Приведите пример различных чистых внешних сил, действующих на одну и ту же систему, вызывая разные ускорения. (b) Приведите пример одной и той же чистой внешней силы, действующей на системы разной массы, вызывая разные ускорения. (c) Какой закон точно описывает оба эффекта? Сформулируйте это словами и в виде уравнения.

8. Если ускорение системы равно нулю, не действуют ли на нее внешние силы? А как насчет внутренних сил? Объясни свои ответы.

9. Если к объекту приложена постоянная ненулевая сила, что вы можете сказать о скорости и ускорении объекта?

10. Сила тяжести, действующая на баскетбольный мяч на рисунке 2, игнорируется. Когда сила тяжести принимается во внимание , каково направление чистой внешней силы на баскетбольный мяч – выше горизонтали, ниже горизонтали или все еще горизонтально?

Задачи и упражнения

Вы можете предположить, что данные, взятые с иллюстраций, имеют точность до трех цифр.

1. Спринтер весом 63,0 кг начинает забег с ускорением 4,20 м / с 2 . Какая чистая внешняя сила действует на него?

2. Если спринтер из предыдущей задачи разгоняется с такой скоростью на 20 м, а затем сохраняет эту скорость до конца 100-метрового рывка, сколько у него времени для забега?

3. Пылесос толкает тележку для белья весом 4,50 кг таким образом, чтобы чистая внешняя сила, действующая на нее, составляла 60,0 Н. Вычислите величину ее ускорения.

4. Поскольку астронавты на орбите явно невесомые, необходим умный метод измерения их массы, чтобы отслеживать прирост или потерю массы и корректировать рацион. Один из способов сделать это – приложить известную силу к космонавту и измерить возникающее ускорение. Предположим, что действует чистая внешняя сила 50,0 Н и измеренное ускорение астронавта составляет 0,893 м / с 2 . (а) Рассчитайте ее массу. (b) При приложении силы к космонавту, аппарат, в котором он движется по орбите, испытывает равную и противоположную силу.Обсудите, как это повлияет на измерение ускорения космонавта. Предложите метод, позволяющий избежать отдачи автомобиля.

5. На рис. 3 чистая внешняя сила на газонокосилке 24 кг заявлена ​​равной 51 Н. Если сила трения, препятствующая движению, равна 24 Н, то какая сила F (в ньютонах) – это сила человека косилка? Предположим, косилка движется со скоростью 1,5 м / с при снятии усилия F . Как далеко уйдет косилка до остановки?

6.Те же ракетные салазки, изображенные на рисунке 5, замедляются со скоростью 196 м / с 2 . Какая сила необходима, чтобы вызвать это замедление? Предположим, что ракеты выключены. Масса системы 2100 кг.

Рисунок 5.

7. (a) Если салазки ракеты, показанные на рисунке 6, запускаются с горящей только одной ракетой, какова величина ее ускорения? Предположим, что масса системы составляет 2100 кг, тяга T составляет 2,4 × 10 4 Н, а сила трения, препятствующая движению, известна как 650 Н.б) Почему ускорение не в четыре раза меньше, чем у всех горящих ракет?

Рисунок 6.

8. Какое замедление у ракетных саней, если они останавливаются за 1,1 с со скорости 1000 км / ч? (Из-за такого замедления один испытуемый потерял сознание и временно потерял сознание.)

9. Предположим, двое детей толкают третьего ребенка в повозке горизонтально, но в противоположных направлениях. Первый ребенок прикладывает силу 75,0 Н, второй – 90.{2} \\ [/ latex] при скорости 90,0 км / ч. На этой скорости силы, противодействующие движению, включая трение и сопротивление воздуха, составляют в сумме 400 Н. (Сопротивление воздуха аналогично трению воздуха. Оно всегда противодействует движению объекта.) Какова величина силы, которую мотоцикл оказывает на мотоцикл в обратном направлении. земля, чтобы произвести его ускорение, если масса мотоцикла с водителем составляет 245 кг?

11. Ракетные салазки, показанные на рисунке 8, ускоряются со скоростью 49,0 м / с 2 . Его пассажирский вес 75.0 кг. (а) Рассчитайте горизонтальную составляющую силы, которую сиденье оказывает на его тело. Сравните это с его весом, используя соотношение. (b) Рассчитайте направление и величину общей силы, которую сиденье оказывает на его тело.

Рисунок 8.

12. Повторите предыдущую задачу для ситуации, когда салазки ракеты замедляются со скоростью 201 м / с 2 . В этой задаче силы прилагаются к сиденью и удерживающим ремням.

13. Вес космонавта вместе с его скафандром на Луне составляет всего 250 Н.Сколько они весят на Земле? Какая масса на Луне? На земле?

14. Предположим, масса полностью загруженного модуля, в котором космонавты взлетают с Луны, составляет 10 000 кг. Тяга его двигателей составляет 30 000 Н. (а) Рассчитайте величину ускорения при вертикальном взлете с Луны. б) Может ли он взлететь с Земли? Если нет, то почему? Если бы это было возможно, вычислите величину его ускорения.

Глоссарий

ускорение:
скорость, с которой скорость объекта изменяется за период времени
свободное падение:
ситуация, в которой единственной силой, действующей на объект, является сила тяжести
трение:
сила соприкасающихся объектов друг с другом; Примеры включают шероховатую поверхность и сопротивление воздуха
чистая внешняя сила:
векторная сумма всех внешних сил, действующих на объект или систему; заставляет массу ускоряться
Второй закон движения Ньютона:
чистая внешняя сила F чистая на объекте с массой m пропорциональна и в том же направлении, что и ускорение объекта, a , и обратно пропорциональна массе; математически определяется как [latex] \ mathbf {\ text {a}} = \ frac {{\ mathbf {\ text {F}}} _ {\ text {net}}} {m} \\ [/ latex]
система
определяется границами наблюдаемого объекта или набора объектов; все силы, возникающие извне системы, считаются внешними силами
вес
сила w за счет силы тяжести, действующей на объект массой [латекс] м [/ латекс]; математически определяется как: w = mg , где g – величина и направление ускорения свободного падения

Избранные решения проблем и упражнения

1.265 N

3. 13,3 м / с 2

7. (a) 12 м / с 2 (b) Ускорение не в четверть от того, что было при горящих ракетах, потому что сила трения все еще такая же большая, как при всех горящих ракетах.

9. (a) Система – это ребенок в вагоне плюс вагон.

Рисунок 9

(б)

(c) a = 0,130 м / с 2 в направлении толчка второго ребенка.

(г) a = 0,00 м / с 2

11.(a) 3,68 × 10 3 Н. Эта сила в 5,00 раз превышает его вес. (b) 3750 Н; 11,3º над горизонтом

13. 1,5 × 10 3 Н, 150 кг, 150 кг

Что такое третий закон Ньютона? – Урок

. (0 Рейтинги)

Быстрый просмотр

Уровень оценки: 6 (5-7)

Требуемое время: 1 час

Зависимость урока:

Тематические области: Физические науки, физика

Ожидаемые характеристики NGSS:


Резюме

Студенты знакомятся с третьим законом движения Ньютона: на каждое действие существует равное и противоположное противодействие.Они практикуют определение пар сил действие-противодействие для множества реальных примеров, а также рисуют и объясняют для них упрощенные векторы (стрелки) на диаграммах свободного тела силы, скорости и ускорения. Они также узнают, что инженеры применяют третий закон Ньютона и понимание сил реакции при разработке широкого спектра творений, от ракет и самолетов до дверных ручек, винтовок и систем доставки лекарств. Этот урок является третьим в серии из трех уроков, предназначенных для проведения перед завершающим сопутствующим заданием по завершению этого раздела. Эта инженерная программа соответствует научным стандартам нового поколения (NGSS).

Инженерное соединение

Рассмотрим яблоко, которое, согласно фольклору, упало на голову Исаака Ньютона и вызвало его мысли о гравитации и движении. Гравитация прикладывает направленную вниз силу к стеблю яблока, а стебель прикладывает равную и противоположную силу вверх, чтобы удерживать яблоко в подвешенном состоянии. Когда стебель стал слишком слабым, чтобы применить столь же сильную силу реакции, яблоко рухнуло вниз, к его голове.

Примеры третьего закона движения Ньютона повсеместно встречаются в повседневной жизни. Например, когда вы прыгаете, ваши ноги прикладывают силу к земле, а земля прикладывает равную и противоположную силу реакции, которая толкает вас в воздух. Инженеры применяют третий закон Ньютона при разработке ракет и других снарядов. Во время запуска горящее топливо оказывает направленное вниз усилие, а сила реакции толкает ракету в воздух. В космосе ракета использует свои задние двигатели для движения вперед, что является еще одним примером того, как инженеры используют силы реакции в своих конструкциях.

Цели обучения

После этого урока учащиеся должны уметь:

  • Определите пары сил действие-противодействие.
  • Нарисуйте и объясните упрощенные диаграммы свободного тела, показывающие пары действие-реакция.
  • Сформулируйте и объясните третий закон Ньютона.

Образовательные стандарты

Каждый урок или задание TeachEngineering соотносится с одним или несколькими научными дисциплинами K-12, образовательные стандарты в области технологий, инженерии или математики (STEM).

Все 100000+ стандартов K-12 STEM, охватываемых TeachEngineering , собираются, обслуживаются и упаковываются сетью стандартов достижений (ASN) , проект D2L (www.achievementstandards.org).

В ASN стандарты иерархически структурированы: сначала по источникам; например , по штатам; внутри источника по типу; например , естественные науки или математика; внутри типа по подтипу, затем по классу, и т. д. .

NGSS: научные стандарты нового поколения – наука
Ожидаемые характеристики NGSS

4-ПС3-4. Примените научные идеи для разработки, тестирования и усовершенствования устройства, преобразующего энергию из одной формы в другую.(4 класс)

Вы согласны с таким раскладом? Спасибо за ваш отзыв!

Нажмите, чтобы просмотреть другие учебные программы, соответствующие этим ожиданиям от результатов.
Этот урок посвящен следующим аспектам трехмерного обучения NGSS:
Наука и инженерная практика Основные дисциплинарные идеи Пересекающиеся концепции
Применяйте научные идеи для решения задач проектирования.

Соглашение о выравнивании: Спасибо за ваш отзыв!

Энергия также может передаваться с места на место с помощью электрического тока, который затем может использоваться локально для создания движения, звука, тепла или света. С самого начала токи могли быть созданы путем преобразования энергии движения в электрическую.

Соглашение о выравнивании: Спасибо за ваш отзыв!

Выражение «производить энергию» обычно относится к преобразованию накопленной энергии в желаемую форму для практического использования.

Соглашение о выравнивании: Спасибо за ваш отзыв!

Возможные решения проблемы ограничены доступными материалами и ресурсами (ограничениями). Успешность разработанного решения определяется с учетом желаемых характеристик решения (критериев). Различные предложения по решениям можно сравнивать на основе того, насколько хорошо каждое из них соответствует указанным критериям успеха или насколько хорошо каждое из них учитывает ограничения.

Соглашение о выравнивании: Спасибо за ваш отзыв!

Энергия может передаваться различными способами и между объектами.

Соглашение о выравнивании: Спасибо за ваш отзыв!

Инженеры улучшают существующие технологии или разрабатывают новые.

Соглашение о выравнивании: Спасибо за ваш отзыв!

Большинство ученых и инженеров работают в группах.

Соглашение о выравнивании: Спасибо за ваш отзыв!

Наука влияет на повседневную жизнь.

Соглашение о выравнивании: Спасибо за ваш отзыв!

Ожидаемые характеристики NGSS

МС-ПС2-1.Примените третий закон Ньютона, чтобы найти решение проблемы, связанной с движением двух сталкивающихся объектов. (6-8 классы)

Вы согласны с таким раскладом? Спасибо за ваш отзыв!

Нажмите, чтобы просмотреть другие учебные программы, соответствующие этим ожиданиям от результатов.
Этот урок посвящен следующим аспектам трехмерного обучения NGSS:
Наука и инженерная практика Основные дисциплинарные идеи Пересекающиеся концепции
Применяйте научные идеи или принципы для разработки объекта, инструмента, процесса или системы.

Соглашение о выравнивании: Спасибо за ваш отзыв!

Для любой пары взаимодействующих объектов сила, оказываемая первым объектом на второй объект, равна силе, которую второй объект оказывает на первый, но в противоположном направлении (третий закон Ньютона).

Соглашение о выравнивании: Спасибо за ваш отзыв!

Модели могут использоваться для представления систем и их взаимодействий, таких как входы, процессы и выходы, а также потоков энергии и материи внутри систем.

Соглашение о выравнивании: Спасибо за ваш отзыв!

Использование технологий и любые ограничения на их использование обусловлены индивидуальными или общественными потребностями, желаниями и ценностями; по результатам научных исследований; а также различиями в таких факторах, как климат, природные ресурсы и экономические условия.

Соглашение о выравнивании: Спасибо за ваш отзыв!

Ожидаемые характеристики NGSS

МС-ПС2-2.Запланируйте расследование, чтобы получить доказательства того, что изменение движения объекта зависит от суммы сил, действующих на объект, и массы объекта. (6-8 классы)

Вы согласны с таким раскладом? Спасибо за ваш отзыв!

Нажмите, чтобы просмотреть другие учебные программы, соответствующие этим ожиданиям от результатов.
Этот урок посвящен следующим аспектам трехмерного обучения NGSS:
Наука и инженерная практика Основные дисциплинарные идеи Пересекающиеся концепции
Планируйте расследование индивидуально и совместно, а также в процессе разработки: определите независимые и зависимые переменные и элементы управления, какие инструменты необходимы для сбора данных, как будут регистрироваться измерения и сколько данных необходимо для подтверждения претензии.

Соглашение о выравнивании: Спасибо за ваш отзыв!

Научные знания основаны на логических и концептуальных связях между доказательствами и объяснениями.

Соглашение о выравнивании: Спасибо за ваш отзыв!

Движение объекта определяется суммой действующих на него сил; если общая сила, действующая на объект, не равна нулю, его движение изменится. Чем больше масса объекта, тем больше сила, необходимая для достижения такого же изменения движения.Для любого данного объекта большая сила вызывает большее изменение в движении.

Соглашение о выравнивании: Спасибо за ваш отзыв!

Все положения объектов и направления сил и движений должны быть описаны в произвольно выбранной системе отсчета и произвольно выбранных единицах размера. Чтобы делиться информацией с другими людьми, необходимо также поделиться этим выбором.

Соглашение о выравнивании: Спасибо за ваш отзыв!

Объяснения стабильности и изменений в естественных или спроектированных системах могут быть построены путем изучения изменений во времени и сил в различных масштабах.

Соглашение о выравнивании: Спасибо за ваш отзыв!

Ожидаемые характеристики NGSS

МС-ПС3-5. Сконструируйте, используйте и представьте аргументы в поддержку утверждения о том, что при изменении кинетической энергии объекта энергия передается к объекту или от него.(6-8 классы)

Вы согласны с таким раскладом? Спасибо за ваш отзыв!

Нажмите, чтобы просмотреть другие учебные программы, соответствующие этим ожиданиям от результатов.
Этот урок посвящен следующим аспектам трехмерного обучения NGSS:
Наука и инженерная практика Основные дисциплинарные идеи Пересекающиеся концепции
Сконструировать, использовать и представить устные и письменные аргументы, подкрепленные эмпирическими данными и научными рассуждениями, для поддержки или опровержения объяснения или модели явления.

Соглашение о выравнивании: Спасибо за ваш отзыв!

Научные знания основаны на логических и концептуальных связях между доказательствами и объяснениями.

Соглашение о выравнивании: Спасибо за ваш отзыв!

Когда энергия движения объекта изменяется, неизбежно одновременно происходит какое-то другое изменение энергии.

Соглашение о выравнивании: Спасибо за ваш отзыв!

Энергия может принимать разные формы (например,грамм. энергия в полях, тепловая энергия, энергия движения).

Соглашение о выравнивании: Спасибо за ваш отзыв!

Международная ассоциация преподавателей технологий и инженерии – Технологии
ГОСТ
Калифорния – Наука
  • Примените научные идеи для разработки, тестирования и усовершенствования устройства, преобразующего энергию из одной формы в другую.(Оценка 4) Подробнее

    Посмотреть согласованную учебную программу

    Вы согласны с таким раскладом? Спасибо за ваш отзыв!

  • Сконструируйте, используйте и представьте аргументы в поддержку утверждения о том, что при изменении кинетической энергии объекта энергия передается к объекту или от него.(Оценки 6 – 8) Подробнее

    Посмотреть согласованную учебную программу

    Вы согласны с таким раскладом? Спасибо за ваш отзыв!

  • Примените третий закон Ньютона, чтобы найти решение проблемы, связанной с движением двух сталкивающихся объектов.(Оценки 6 – 8) Подробнее

    Посмотреть согласованную учебную программу

    Вы согласны с таким раскладом? Спасибо за ваш отзыв!

  • Запланируйте расследование, чтобы получить доказательства того, что изменение движения объекта зависит от суммы сил, действующих на объект, и массы объекта.(Оценки 6 – 8) Подробнее

    Посмотреть согласованную учебную программу

    Вы согласны с таким раскладом? Спасибо за ваш отзыв!

Предложите выравнивание, не указанное выше

Какое альтернативное выравнивание вы предлагаете для этого контента?

Рабочие листы и приложения

Посетите [www.teachengineering.org/lessons/view/ucd_newton_lesson03], чтобы распечатать или загрузить.

Больше подобной учебной программы

Ньютон заставляет меня двигаться

Учащиеся изучают движение, ракеты и движение ракет, помогая космонавту Тесс, космонавту Рохану и Майе в их исследованиях. Сначала они узнают некоторые основные факты о транспортных средствах, ракетах и ​​о том, почему мы их используем. Затем они обнаруживают, что движение всех объектов, включая полет ракеты и движения…

Что такое первый закон Ньютона?

Студенты знакомятся с концепциями силы, инерции и первого закона движения Ньютона: объекты в состоянии покоя остаются в состоянии покоя, а объекты в движении остаются в движении, если на них не действует несбалансированная сила. Студенты узнают разницу между скоростью, скоростью и ускорением и приходят к пониманию того, что ча…

Забери меня с этой планеты

Цель этого урока – научить студентов, как космический корабль попадает с поверхности Земли на Марс. Студенты сначала исследуют ракеты и то, как они могут доставить нас в космос. Наконец, обсуждается природа орбиты, а также то, как орбиты позволяют нам перемещаться с планеты на планету – спец…

Что такое второй закон Ньютона?

Студенты знакомятся со вторым законом движения Ньютона: сила = масса x ускорение. Обсуждаются как математическое уравнение, так и физические примеры, в том числе машина Этвуда для иллюстрации принципа. Студенты приходят к пониманию того, что ускорение объекта зависит от его массы и силы удара…

Предварительные знания

Студенты должны быть знакомы с понятиями массы, свойств материи (веса, плотности, объема) и основных алгебраических уравнений.

Введение / Мотивация

Третий закон движения Ньютона основывается на первом и втором законах движения.Третий закон движения гласит, что на каждое действие существует равное и противоположное противодействие. Это можно наблюдать как в покоящихся, так и в ускоряющихся объектах. Например, лежащий ящик толкается о землю под действием силы тяжести. В ответ на это земля с такой же силой давит вверх, что мы называем «нормальной силой». Эти силы уравновешиваются, поэтому ускорение коробки не происходит. авторское право

Copyright © (дверная ручка) 2014 Дениз В. Карлсон, Инженерный колледж, Университет Колорадо в Боулдере; (самолет) 2004 Microsoft Corporation, One Microsoft Way, Redmond, WA 98052-6399 USA.Все права защищены. http: //office.microsoft.com/en-us/images/results.aspx? qu = airplane & ex = 1 # ai: MP2454 | mt: 2 |

Третий закон Ньютона можно также соблюдать в ракетах и ​​других снарядах. Для запуска большая сила создается двигателями ракеты в пространстве позади нее. В ответ на эту силу воздух отталкивается с равной силой, толкая ракету вперед. Какие еще примеры вы можете придумать?

(Продолжите, показав презентацию и доставив контент в разделе «Предпосылки урока».)

Предпосылки и концепции урока для учителей

Подготовка учителей

  • Будьте готовы показать студентам презентацию о силах и третьем законе Ньютона (презентация PowerPoint® с семью слайдами), чтобы провести урок.
  • Для демонстрации «Двигатель героя» возьмите под рукой банку с газировкой, гвоздь, кусок веревки и воду. Гвоздем проделайте четыре дырочки в основании пустой банки из-под газировки. Когда гвоздь находится в каждом отверстии, толкайте гвоздь влево или вправо, чтобы наклонить отверстие так, чтобы вода текла по касательной, создавая толчок; проделайте то же самое для каждой лунки.Привяжите веревку к язычку. Для демонстрации наполните банку водой и поднимите ее за веревку над раковиной или ванной (или снаружи), чтобы ученики могли наблюдать вращательное движение банки, вращающейся по мере того, как вода вытекает из отверстий. См. Инструкции на слайде 2 и в NASA Pop Can «Hero Engine».
  • Заранее сделайте копии заключительной викторины Ньютона (по одной на каждого учащегося).
  • В какой-то момент во время презентации, возможно, говоря о некоторых примерах действия-противодействия (слайды 3-6), переходите к тому, как рисовать (концептуальные) векторы (стрелки) диаграммы свободного тела силы, скорости и ускорения, которые учащиеся будет предложено сделать как часть домашнего задания.

Третий закон движения Ньютона Схема презентации (слайды 1-7)

Откройте презентацию «Силы и третий закон Ньютона», чтобы все студенты могли просмотреть и представить содержание урока, руководствуясь приведенным ниже сценарием и текстом в примечаниях к слайду. Слайды анимированы, поэтому при нажатии отображается следующий текст / изображение / ответ.

Цель: уметь идентифицировать пары сил действие-противодействие .

( слайд 2 ) Представьте третий закон движения Ньютона: для каждого действия существует равная и противоположная реакция.Спросите студентов: слышали ли вы это раньше? Как вы думаете, что это значит?

Затем продемонстрируйте третий закон, показав студентам современную версию Hero’s Engine, что занимает всего несколько минут. Герой Александрии был древнегреческим математиком и экспериментатором, жившим в Египте. Его первоначальный двигатель был паровым, но версия из банки с газировкой хорошо демонстрирует ту же концепцию. Для демонстрации наполните подготовленную банку водой и поднимите ее за веревку над раковиной или ванной (или снаружи), чтобы учащиеся могли наблюдать вращательное движение, когда вода вытекает из отверстий и банка вращается.Банка вращается из-за силы реакции, связанной с потоком воды.

Как вариант, продемонстрируйте третий закон, предложив одному ученику сесть на самокат с баскетбольным мячом, а затем бросить мяч другому ученику. Сила реакции от броска очевидна, когда бросающий ученик отталкивается назад на самокате.

(, слайд 3 ) Сообщите учащимся, что силы возникают в так называемых «парах сил действие-противодействие». На этом слайде показана стопка бетонных блоков, лежащих на земле.Определите пару действие-реакция для класса: вес блока давит на землю, а земля толкает обратно вверх на блок.

( слайд 4 ) Затем попросите учащихся определить пару действие-реакция на фотографии стреляющей пушки. Пушка воздействует на пушечное ядро, а пушечное ядро ​​оказывает на пушку равную и противоположную силу. Обратите внимание на то, что третий закон Ньютона объясняет отдачу метательного оружия, такого как пушки и ружья. Учащиеся, которые видели Wall-E, могут вспомнить сцену, в которой робот использует огнетушитель в качестве силовой установки (сила реакции заставляет робота двигаться).Еще один хороший пример третьего закона / отдачи – садовый шланг, танцующий по двору, движущийся из-за силы воды, протекающей через него.

( слайд 5 ) Попросите учащихся определить пары сил действие-противодействие на фотографии запуска космического челнока. Космический шаттл оказывает направленное вниз усилие, а сила реакции толкает его вверх.

( слайд 6 ) Попросите учащихся определить все пары сил действие-противодействие на этой фотографии двух футболистов.Примеры: ручной шлем, рука-плечо, рука с мячом, площадка для обуви.

Возможно, сейчас самое время рассмотреть, как рисовать (концептуальные) векторы (стрелки) диаграммы свободного тела силы, скорости и ускорения.

( слайд 7 ) Просмотрите концепции всех трех уроков этого модуля . Завершите презентацию обзором ключевых понятий, перечисленных на слайде, с пробелами, чтобы учащиеся могли дать ответы. Ожидайте, что благодаря этим трем урокам учащиеся научатся понимать три закона движения Исаака Ньютона.Эти фундаментальные законы физики описывают, как силы влияют на движение объектов. Без сил не может произойти никаких изменений в движении. Понимание сил может быть очень мощной вещью! Поскольку инженеры понимают, как силы заставляют объекты замедляться, ускоряться и поворачиваться, они могут проектировать сложные механические системы, от самолетов до дверных ручек и тонких систем доставки лекарств.

Затем проведите соответствующее задание «Раздвижные учебники», а затем выполните заключительную викторину, как описано в разделе «Оценка».

Сопутствующие мероприятия

  • Раздвижные учебники – в эксперименте, состоящем из двух частей, с использованием стола, учебника, веревки и бутылки с водой, студенческие команды исследуют, как силы, такие как приложенная сила и трение, влияют на ускорение объекта. Они составляют график и анализируют свои результаты, рассматривают применения для поверхностей с высоким и низким коэффициентом трения и видят, как силы играют роль в инженерном проектировании.

Словарь / Определения

ускорение: величина изменения скорости объекта.

сила: толкание, притяжение или скручивание объекта.

инерция: сопротивление объекта изменению своего движения.

Первый закон Ньютона: если на объект не действует неуравновешенная сила, неподвижный объект остается в покое, а объект в движении остается в движении.

Второй закон Ньютона: Сила = масса x ускорение, также известное как F = ma

Третий закон Ньютона: на каждое действие существует равное и противоположное противодействие.

скорость: скорость и направление объекта.

Оценка

Оценка перед уроком

Домашнее задание Обзор: Просмотрите ответы студентов на Первый и Второй законы Ньютона Домашнее задание , которое было задано в конце предыдущего урока, Что такое Второй закон Ньютона? Убедитесь, что студенты уверены в первом и втором законах Ньютона, прежде чем переходить к третьему закону Ньютона.

Оценка после введения

Вопросы: В качестве встроенной оценки оцените понимание учащимися третьего закона Ньютона на основе их ответов на вопросы на слайдах 4, 5 и 6 презентации «Силы и третий закон Ньютона». Используйте вопросы на слайде 7 в качестве обзора перед проведением финальной викторины.

Итоги урока Оценка

Тест модуля: После просмотра вопросов на слайде 7, ответа на все оставшиеся вопросы учащихся и выполнения соответствующего задания, скользящего учебника, проведите заключительный тест по законам Ньютона в качестве оценки, охватывающей материал всех трех уроков модуля.Для этого учащиеся должны нарисовать (концептуальные) векторы (стрелки) диаграммы свободного тела силы, скорости и ускорения. Как вариант, проведите тест после этого урока (перед тем, как проводить соответствующее задание).

Рекомендации

Лувьер, Грузия. «Законы движения Ньютона». 2006. Университет Райса. По состоянию на 1 апреля 2014 г. http://teachertech.rice.edu/Participants/louviere/Newton/index.html

«Законы Ньютона.”2014. Physics Tutorial, The Physics Classroom. Проверено 1 апреля 2014 г. http://www.physicsclassroom.com/class/newtlaws

.

Авторские права

© 2014 Регенты Университета Колорадо; оригинал © 2013 Калифорнийский университет в Дэвисе

Авторы

Элизабет Энтони; Скотт Штробель; Яков Тетер

Программа поддержки

Программа RESOURCE GK-12, Инженерный колледж, Калифорнийский университет в Дэвисе

Благодарности

Содержание этой учебной программы в цифровой библиотеке было разработано в рамках проекта «Возможности систем возобновляемой энергии для унифицированного научного сотрудничества и образования» (RESOURCE) Инженерного колледжа в рамках гранта GK-12 Национального научного фонда.DGE 0948021. Однако это содержание не обязательно отражает политику Национального научного фонда, и вам не следует предполагать, что оно одобрено федеральным правительством.

Последнее изменение: 15 мая 2021 г.

5.3 Второй закон Ньютона – Университетская физика, том 1

Какая ракетная тяга ускоряет этот снегоход?
До космических полетов с участием космонавтов ракетные сани использовались для проверки самолетов, ракетного оборудования и физиологических воздействий на людей на высоких скоростях.Они состояли из платформы, которая была установлена ​​на одной или двух направляющих и приводилась в движение несколькими ракетами.

Рассчитайте величину силы, прилагаемой каждой ракетой, которая называется ее тягой T , для четырехракетной двигательной установки, показанной на рис. 5.14. Начальное ускорение салазок составляет 49 м / с 249 м / с2, масса системы 2100 кг, сила трения, препятствующая движению, составляет 650 Н.

Рис. 5.14. Салазки испытывают ракетную тягу, которая ускоряет их вправо. Каждая ракета создает одинаковую тягу Т .Система здесь – это сани, его ракеты и его всадник, поэтому никакие силы между этими объектами не учитываются. Стрелка, обозначающая трение (f →) (f →), нарисована больше масштаба.
Стратегия
Хотя силы действуют как вертикально, так и горизонтально, мы предполагаем, что вертикальные силы компенсируются, потому что нет вертикального ускорения. Это оставляет нам только горизонтальные силы и более простую одномерную задачу. Направления указываются знаками плюс или минус, при этом вправо принимается за положительное направление.См. Диаграмму свободного тела на рис. 5.14.
Решение
Поскольку ускорение, масса и сила трения даны, мы начнем со второго закона Ньютона и ищем способы найти тягу двигателей. Мы определили направление силы и ускорения как действующее «вправо», поэтому в расчетах нам нужно учитывать только величины этих величин. Следовательно, мы начинаем с

, где FnetFnet – чистая сила в горизонтальном направлении. Из рисунка видно, что тяга двигателя увеличивается, а трение противодействует тяге.В форме уравнения чистая внешняя сила равна

.

Подставляя это во второй закон Ньютона, получаем

Fnet = ma = 4T − f. Fnet = ma = 4T − f.

Используя небольшую алгебру, мы решаем общую тягу 4 T :

Подстановка известных значений дает

4T = ma + f = (2100 кг) (49 м / с2) + 650N. 4T = ma + f = (2100 кг) (49 м / с2) + 650N.

Следовательно, общая тяга

4T = 1.0 × 105N, 4T = 1.0 × 105N,

и индивидуальные тяги

T = 1.0 × 105N4 = 2.5 × 104N, T = 1.0 × 105N4 = 2.5 × 104N.
Значение
Цифры довольно большие, поэтому результат может вас удивить.Подобные эксперименты проводились в начале 1960-х годов, чтобы проверить пределы человеческой выносливости, и установка была разработана для защиты людей при аварийных выбросах реактивных истребителей. Была получена скорость 1000 км / ч с ускорением 45 g ‘с. (Напомним, что g , ускорение свободного падения составляет 9,80 м / с 29,80 м / с2. Когда мы говорим, что ускорение составляет 45 g , это 45 × 9,8 м / с2,45 × 9,8 м / с2. , что составляет примерно 440 м / с – 2440 м / с2.) Хотя живые предметы больше не используются, с помощью ракетных саней была получена сухопутная скорость 10 000 км / ч.

В этом примере, как и в предыдущем, интересующая система очевидна. В последующих примерах мы увидим, что выбор интересующей системы имеет решающее значение, и этот выбор не всегда очевиден.

Второй закон Ньютона – это больше, чем определение; это соотношение между ускорением, силой и массой. Это может помочь нам делать прогнозы. Каждую из этих физических величин можно определить независимо, поэтому второй закон говорит нам что-то основное и универсальное о природе.

.

Оставить комментарий